Legal Fund MBE Study Set

Ace your homework & exams now with Quizwiz!

A girlfriend and boyfriend decide they can make some money by setting up a bomb to cause minor damage in a bank and then demand payment with the threat of several other bombs which are planted in the bank. However, the boyfriend decides that minor damage won't make enough of an impression and beefs up the strength of the bomb. They place the bomb together, and when it is detonated, a bank teller is killed. What is the most serious crime the girlfriend could be guilty of? (A) Felony murder. (B) Robbery. (C) Conspiracy to commit murder. (D) Murder.

The correct answer is: (A) Felony murder. Felony murder is an unintentional killing proximately caused during the commission or attempted commission of a serious or inherently dangerous felony. A co-felon is responsible for all actions his partners in crime take in furtherance of the crime, even if he does not have complete knowledge of or did not participate directly in those acts. In this case, the boyfriend and the girlfriend were planning on setting off a bomb in a bank only as a threat to demand money, in essences to commit a robbery, which is an inherently dangerous crime. However, even though the bomb was meant only as a threat and not intended to kill someone, the bank teller was killed as a result of the robbery, which is felony murder. Furthermore, although it was the boyfriend that decided to make the bomb stronger, which directly resulted in the death of the bank teller, the act was committed in furtherance of the crime the two were perpetrating together, and thus the girlfriend is liable for the results of that action and can be convicted of felony murder.

The owner of a 23-hour convenience store hired his friend as a sales clerk. However, when the owner noticed that large amounts of merchandise were disappearing during his friend's shift at the store, he fired his friend without offering him any chance to explain. They now play on a softball team together. During the final game of the season, while trying to make a double-play, the former clerk accidentally hit the store owner with the softball. The store owner was furious, convinced that his former clerk had acted intentionally, in retaliation for being fired. After the game, the store owner waited in the parking lot, sipping whisky from his ever-present flask. When he saw his former clerk walking toward his car, the store owner charged at him with a baseball bat, saying, "You just played your last game!" He pummeled his former clerk and left him lying there unconscious. Later, another player on the team called the store owner and told him, "Your friend is dead!" The store owner responded, "Good!" Of what crime is the store owner guilty? (A) First-degree murder. (B) Second-degree murder. (C) Felony murder. (D) Involuntary manslaughter.

The correct answer is: (A) First-degree murder. The crime known as first-degree murder actually encompasses several types of homicide: intent-to-kill murder committed with premeditation and deliberation; felony murder; and in some jurisdictions, murder accomplished by lying in wait, poison or torture. If a murderer engages in any reflection or premeditation, even if the reflection is cursory or brief, he may be guilty of first-degree murder. Here, the facts indicate that the store owner intended not merely to injure his former clerk but to end his life, as evidenced by his remark, "You just played your last game!" The store owner's response to the news that his former friend was dead also indicates an intent to kill him. The facts further indicate that prior to committing the homicide, the store owner engaged in reflection and premeditation as he waited in the parking lot. And while extreme intoxication may, at times, negate a killer's premeditation and deliberation, it is unlikely that the store owner's consumption of alcohol in the parking lot was sufficient to negate his premeditation. Thus, all the elements of intent-to-kill murder are present, and the store owner is guilty of the crime of first-degree murder.

An earnest father, inspired by an Internet advertisement for a new roller coaster, drives his family cross- country to the "newest and best" amusement park anywhere. According to the advertisements, the park's new Star Safari roller coaster makes all other roller coasters obsolete. The ad stated that the ride would open to the general public on July 1. However, when the father and his family arrive at the amusement park on August 1, they are informed that the Star Safari is not yet open to the general public; first, the park is offering exclusive rides to those on its special guest list. The rest of the family decides to make the best of the situation and leaves to enjoy the other rides at the park, but the father is furious and demands admission to the Star Safari. After a heated exchange with the park manager, the father is escorted out of the park and told that he is banned for life. On his way out, the irate father yells, "If this is the way you treat your customers, I'll make sure that no one ever gets to use that ride!" Just before he exits the park, the father grabs a woman and her young son, forces them into his car, and drives them back to his hotel. Once back at the hotel, the father rents a second room and locks his hostages inside. He then calls the woman's husband and tells him that unless the husband plants a bomb, to be supplied by the father, at the base of the Star Safari ride, the father will kill the hostages. The panicked husband complies and comes to pick up the bomb from the father. The father informs the husband of the woman hostage and that the bomb is set to detonate during the early morning, prior to the park's opening, so that only the ride itself will be damaged. However, in setting the timer on the bomb, the father forgot that he was now in a different time zone, three hours later than his accustomed time zone. As such, the bomb detonates not at 6:00 a.m. but at 9:00 a.m., just after the Star Safari's first run of the day. Numerous people are injured, and three are killed. If the husband who placed the bomb is tried for felony murder, what is the likely verdict? (A) Not guilty, because duress is a defense to the setting of an explosive device. (B) Not guilty, because the bomb went off at a different time than he expected. (C) Guilty, because the people's deaths occurred during the course of the husband's commission of a felony. (D) Guilty, because duress is not a defense to murder.

The correct answer is: (A) Not guilty, because duress is a defense to the setting of an explosive device. Felony murder is an unintentional killing proximately caused during the commission or attempted commission of any of several named serious or inherently dangerous felonies. In this case, while the setting of an explosive device most likely falls within the definition of an inherently dangerous felony, the husband acted under extreme duress--that is, the father's threat to kill the husband's wife and child. Because duress is a defense to the setting of an explosive device, the husband is not guilty of that felony. In addition, unless a defendant is guilty of the independent underlying felony, he cannot be convicted of felony murder. As such, although people did die as the result of the husband's act, the husband cannot be found guilty of felony murder (although he may be guilty of another form of homicide).

The defendant and his wife are separated. The wife recently began dating again. She tells the defendant that the relationship is moving along quickly and that she is happy. One evening, the wife brings her new boyfriend to the bar and restaurant the defendant owns. Although the wife does not intend to throw her happiness in her husband's face, the defendant takes enormous offense and feels publicly humiliated that his wife has brought a date to his own place of business. The defendant approaches his wife and the boyfriend to air his feelings, and an argument breaks out. After several minutes, the defendant orders his wife and the victim to "get out!" The defendant then starts to walk way, but the boyfriend comes up behind him and grabs the defendant by the arm. The boyfriend sticks his finger in the defendant's face and tells him to apologize to his wife immediately. The defendant attempts to brush him off. When the boyfriend makes a fist, the defendant grabs a steak knife from the nearest table and rams it into the boyfriend's gut. The boyfriend is rushed to the hospital and into surgery. During the surgery, the boyfriend's heart gives out due to a preexisting heart condition that makes surgical anesthesia a problem for him, and he dies. Doctors now agree that the boyfriend would most likely have recovered from the knife wound without surgery. The defendant is charged with murder. What is the defendant's best defense? (A) The defendant acted in the heat of passion. (B) The defendant didn't intend to cause the victim's death. (C) The boyfriend would have survived the knifing if the doctors had not opted for immediate surgery. (D) The defendant didn't know that the boyfriend had a heart condition.

The correct answer is: (A) The defendant acted in the heat of passion. Voluntary manslaughter, often called "heat-of-passion" killing, is an intentional killing mitigated by adequate provocation or other circumstances negating malice aforethought. The classic examples of adequate provocation are instances where a person is the victim of a serious battery or threat of a deadly force or finds his/her spouse engaged in sexual conduct with another person. Here, the circumstances are consistent with a finding that the defendant stabbed the boyfriend in the heat of passion, mitigating the defendant's criminal intent and thereby reducing the defendant's crime from murder to voluntary manslaughter. While the defendant did not discover his wife and the boyfriend engaged in sexual relations, the circumstances are similar in that the defendant encountered his wife and her boyfriend on a date in his own restaurant. In addition, the fact that the boyfriend had grabbed the defendant, ordered him to apologize to the wife, and raised a fist to him supports the argument that the defendant was responding to a threat of battery. Thus, the combination of apparent marital infidelity and threat of physical injury make the defendant's defense that he acted in the heat of passion potentially viable.

A builder from town owned Blueacre, bordered on the north by a dirt road called Rocky Road. An investor owned Redacre, directly to the south of Blueacre. Redacre was bordered on its south side by a river. There were no buildings on either lot. The investor and the builder lived in town and rarely visited their respective properties. The builder sold Blueacre to a farmer. The deed from Builder to Farmer described Blueacre as extending "from Rocky Road on the north to the river on the south." Farmer, a friend of the investor, noticed the error in the deed but used the same description two years later in conveying Blueacre to an Excavator. The Excavator believed in good faith that Blueacre extended from Rocky Road to the river. Excavator built a quarry on the property that extended partially onto Redacre, living and working there for 20 years until he sold the property to a young man, using the same description of the property as had been used in the deed from Farmer to Excavator. The young man made plans to expand the quarry's operations toward the river, intending to use the river to dump sludge and to transport gravel from the quarry. A month later, and before the young man expanded, the investor discovered the quarry for the first time. The investor brings an action to eject the young man from Redacre, and to quiet title to his property. Assuming the relevant statutory period is 20 years, who is most likely to prevail? (A) The young man, under the doctrine of constructive adverse possession. (B) The young man, but only regarding the portion of Redacre the quarry occupies. (C) The investor, because the young man's possession was not sufficiently hostile. (D) The investor, because the young man's possession of Redacre did not last for 20 years.

The correct answer is: (A) The young man, under the doctrine of constructive adverse possession. A person may acquire title to property through adverse possession if, for the statutory period, his possession is (1) open, visible and notorious, (2) actual, (3) exclusive, (4) continuous, and (5) hostile under a claim of right. If an adverse possessor has not been in possession of the piece of land at issue for the full statutory period, he may still meet the time period by tacking his possession onto possession by a previous possessor, if the subsequent possessor is in privity with the prior possessor by deed, devise, or descent. The doctrine of constructive adverse possession comes into play when the deed is defective, as here, and satisfies the actual possession element. The doctrine requires color of title (e.g., a defective deed) and actual possession of at least a significant portion of the property. Subject to exceptions inapplicable here, when one possesses a portion of a disputed tract of land, his adverse possession is sufficient to give him title to the entire tract of land, so long as there is a significant relationship between the portion of the tract actually possessed and the remainder of. The young man, as well as the previous owner, Excavator, openly claimed title to the northern portion of Redacre based ontheir operation of a quarry on that land. The presence of the quarry meets each of the requisite elements of adverse possession. Both Excavator and the young man's continued presence on the land meets the 20 year statutory requirement. Since the young man meets all of the requirements for adverse possession and may tack his holding period onto Excavator's, the young man will prevail as to the whole of Redacre under the doctrine of constructive adverse possession.

A doctor loaned his neighbor his motorcycle so the neighbor could make a good impression for a first date. The neighbor and the doctor were citizens of State A and worked in that state. The neighbor told the doctor that he was going to only drive the motorcycle locally. The neighbor and his date were having so much fun that they decided to go to a casino in State Y. The neighbor became involved in a traffic accident with a student while driving in State Y. The student brought an action against the doctor in the proper U.S. District Court in State Y for personal injuries and property damages in the amount of $85,000. A State Y statute permits an action against the owner of a motor vehicle where he/she loaned the car to the person who was driving it when the incident occurred. If the doctor makes a motion to dismiss the action for lack of personal jurisdiction, will the court grant the motion? (A) Yes, because the assertion of personal jurisdiction over him would probably violate due process. (B) Yes, unless State Y had a long-arm statute that specifically authorized service of process upon an out-of-state citizen in circumstances such as these. (C) No, because federal courts have nationwide service of process. (D) No, because the State Y law specifically extends liability to the doctor in circumstances such as these.

The correct answer is: (A) Yes, because the assertion of personal jurisdiction over him would probably violate due process. In the absence of a traditional basis for obtaining personal jurisdiction, the assertion of personal jurisdiction over an out-of-state citizen must be consistent with due process (i.e., the defendant must have sufficient minimum contacts with the forum as to not offend traditional notions of fair play and substantial justice). Since the doctor could not foresee being haled into State Y (The neighbor had misrepresented to the doctor that his trip would terminate in State A), and apparently has no other contacts with State Y, the assertion of personal jurisdiction over the doctor would probably violate due process.

A plaintiff sued a defendant in the United States District Court for the District of State A. The claim arose from a contract that was negotiated, made, and allegedly breached in State B. The long-arm statute of State A contained a quasi-in-rem provision, which provided that personal jurisdiction over a defendant could be obtained by the seizure of a defendant's personal property in State A, regardless of whether the property bore any relation to the suit. The statute further provided that the limit of the court's jurisdiction would be to enter a money judgment for the value of such seized property. The plaintiff had the defendant's property seized while it was being transported by truck through State A. The property bore no relation to the dispute. The defendant was incorporated and had its principal place of business in State C and had no other contacts with State A. The defendant filed a timely motion to dismiss for lack of personal jurisdiction. The court granted the motion. Was the court correct to grant the motion to dismiss for lack of personal jurisdiction? (A) Yes, because the defendant did not have minimum contacts with the forum state. (B) Yes, because quasi-in-rem jurisdiction was unconstitutional. (C) No, because the plaintiff complied with the quasi-in-rem statute by seizing the defendant's property. (D) No, because the defendant had sufficient "minimum contacts" with State A because it shipped its property through State A by truck.

The correct answer is: (A) Yes, because the defendant did not have minimum contacts with the forum state. In Shaffer v. Heitner, the Supreme Court of the United States held that all assertions of quasi-in-rem jurisdiction must be evaluated according to the minimum contacts analysis of International Shoe v. Washington and its progeny. In this case, there could have been no general jurisdiction over the defendant because it was incorporated and had its principal place of business elsewhere. Nor could there have been specific jurisdiction, because the sending of the property through State A had nothing to do with the related claim. There were no minimum contacts sufficient to hale the defendant into the courts of State A.

A plaintiff was injured when her automobile tire blew out while she was driving in a foreign country. The plaintiff sued two defendants, the manufacturer of the tire and the distributor of the tire. The plaintiff brought the suit in the United States District Court for the Northern District of State A, where the defendant manufacturer had its principal place of business. The defendant distributor had its principal place of business in the Southern District of State A. Diversity jurisdiction existed. The defendants filed a motion to dismiss based upon the doctrine of forum non conveniens. The court granted the motion because the foreign country would be a much more convenient forum for the litigation. Was the court correct when it granted the motion to dismiss? (A) Yes, because the foreign country was a much more convenient forum. (B) Yes, because venue was improper in the Northern District of State A. (C) No, because the court had personal jurisdiction over both defendants. (D) No, because the court had subject matter jurisdiction.

The correct answer is: (A) Yes, because the foreign country was a much more convenient forum. The doctrine of forum non conveniens is about the relative convenience of the forum where a case is brought and an alternative forum in another court system, such as another state or a foreign country. Here, the plaintiff sued in a forum where personal jurisdiction, subject matter jurisdiction, and venue all were proper, but because the foreign country where the accident happened would have been a much more convenient forum the court nevertheless had the power to dismiss the case under the forum non conveniens doctrine, in contemplation of the case being brought in that more convenient forum.

A dog breeder loses her prized golden retriever puppy during a walk in a nature preserve. To enlist help in finding her dog, the breeder posts a notice on the internet and offers to pay $1000 for the return of her puppy. A hiker sees the notice, equips herself, drives to the nature preserve and finds the puppy. Before the hiker can return the puppy, the breeder hears that the dog has been found and posts another notice on the internet that withdraws the reward offer. The hiker brings the breeder the dog and demands the reward. The breeder refuses. Is the breeder liable to pay the hiker the reward? (A) Yes, because the hiker had undertaken substantial steps toward acceptance of the breeder's offer before it was revoked. (B) Yes, because the hiker did not actually see the breeder's purported revocation of her reward offer. (C) No, because the hiker did not communicate her acceptance of the breeder's offer. (D) No, because the breeder revoked the offer before completion of the hiker's performance.

The correct answer is: (A) Yes, because the hiker had undertaken substantial steps toward acceptance of the breeder's offer before it was revoked. A unilateral contract is created when an offer seeks performance rather than a promise in return. An offer in a unilateral contract cannot be revoked once performance has begun. This answer is correct because the hiker undertook substantial efforts towards acceptance by performance of the breeder's offer to enter into a unilateral contract. All that remained was delivery of the puppy to the breeder. Therefore, the breeder cannot revoke her offer to enter into a unilateral contract once the hiker began performance.

A plaintiff sued a defendant in federal court in State A under diversity jurisdiction. The defendant was a citizen of State B and did not derive at least $500,000 in annual revenue from sales in State A. The plaintiff alleged that the defendant caused injury to the plaintiff by an act in State B that harmed the plaintiff in State A. The long-arm statute of State A empowered the courts in State A to reach out-of-state defendants who caused injury in the state by an act or omission outside the state only if the defendant derived at least $500,000 in annual revenue from sales in State A. The defendant filed a timely motion to dismiss for lack of personal jurisdiction. The court granted the motion. Was the court correct to grant the motion to dismiss? (A) Yes, because the long-arm statute of the forum state did not reach the defendant. (B) Yes, because the act that allegedly gave rise to liability occurred outside the forum state. (C) No, because it would have been constitutional for State A to assert personal jurisdiction over the defendant under these circumstances. (D) No, because the state long-arm statute was not relevant in federal court.

The correct answer is: (A) Yes, because the long-arm statute of the forum state did not reach the defendant. A federal court sitting in diversity may reach an out-of-state defendant only in accordance with Federal Rule of Civil Procedure 4(k)(1)(A), which makes the reach of the federal court coextensive with the reach of state courts in the forum state. Here, the state long-arm statute did not reach the defendant. Therefore, there was no personal jurisdiction in the federal court regardless of whether such an assertion of jurisdiction would have been constitutional.

A skier purchased a parcel of land from a landowner when she was 30 years old. The skier intended to develop the parcel as a ski resort. The next year, however, the skier suffered a head injury and was declared legally incompetent before she could make any improvements to the land. Five years later, a snowboarder purchased the parcel next to the skier's parcel. The snowboarder built a large ski resort. A few years later, when the injured skier was 40 years old, the snowboarder inadvertently expanded her resort onto the skier's land. The skier never recovered from her head injury and died at age 61. The skier's daughter inherited the skier's estate. Five years after the skier's estate passed to the skier's daughter, the snowboarder was killed by a falling gondola. The snowboarder's children, a brother and a sister, inherited the resort as joint tenants. That same year, the skier's daughter sought to eject the brother and the sister from that part of their resort that was on the daughter's land. The statutory period for adverse possession is 20 years. Will the daughter prevail in her suit against the brother and the sister? (A) Yes, because the statute period was tolled during the incompetency of the skier. (B) Yes, because the brother and the sister have not been in possession for 20 years. (C) No, because the brother and the sister possess color of title to the daughter's entire parcel. (D) No, because the brother and the sister's period of possession will be tacked onto the snowboarder's period of possession.

The correct answer is: (A) Yes, because the statute period was tolled during the incompetency of the skier. In order to gain title to real property adversely, one's possession must be: a) open, visible, and notorious; b) actual; c) exclusive; d) hostile and under claim of right or title; and e) continuous for the statutory period. The statute of limitations is tolled (not started) if a disability exists when the adverse possessor enters the property. The statutory period begins when the disability is removed. In this case, the snowboarder's statutory period of adverse possession did not begin until the daughter inherited the land. Prior to that, the record owner of the property, the skier, was legally incompetent and this disability was not removed until she died and the daughter inherited. Therefore, the statutory period had yet to end when the daughter sought to eject the brother and the sister. Consequently, the brother and the sister did not gain title to any of the daughter's land by means of adverse possession.

A plaintiff sued two defendants in state court in State A. The plaintiff alleged a cause of action under federal law against the first defendant and a claim that arose under state law against the second defendant. All parties were citizens of State A. All claims arose from the same events. The defendants removed the case to the United States District Court for the District of State A. The plaintiff moved to remand the case back to state court. The federal court denied the motion to remand. Was the federal court correct when it denied the motion to remand? (A) Yes, because there was a federal question in the case. (B) Yes, because the case could have been brought originally in federal court under diversity of citizenship. (C) No, because the defendants were citizens of State A. (D) No, because there was no independent basis of jurisdiction over the claim against the second defendant.

The correct answer is: (A) Yes, because there was a federal question in the case. The first issue for a removal question is whether the case could have been brought originally in federal court. The answer here was yes. Federal question jurisdiction existed over the federal claim against the first defendant. Supplemental jurisdiction existed over the claim against the second defendant because the claim arose from the same events as the federal claim against the first defendant. The next questions to ask are whether the case was founded solely on diversity and, if so, whether one of the defendants was local (a citizen of the state where the case was brought). Here, the case was not founded solely on diversity there was a federal question in the case and thus the limits on removal that apply to diversity cases do not apply here.

A woman owned a used bookstore. She was passionate about literature, and spent most of her days behind the counter reading, barely glancing up at customers as they came and went. She had a high employee turnover rate, as her employees usually got frustrated with her lack of management, and, therefore, the general upkeep of the shop was in decline. In fact, there were several cracks in the roof of the shop; over the past few weeks rain had leaked in and formed puddles on the floor, which the bookstore owner noticed but failed to clean. One day a customer came into the store drinking a large soda. He carelessly dropped it on the floor, and the bookstore owner failed to clean it. About four hours later, a second customer came into the bookstore, hoping to find a first-edition of a book. The second customer, trying to break in a new pair of 5-inch heels that were a size too small, could barely walk. As she was teetering up and down the aisles browsing the selection, she slipped on the wet floor and broke her ankle. If the second customer sues the bookstore owner for negligence, why would she prevail? (A) Because she sued in a contributory negligence jurisdiction, and the jury found her less responsible than the bookstore owner. (B) Because she sued in a pure comparative negligence jurisdiction, and the jury found her 90% responsible and found the bookstore owner only 10% responsible. (C) Because she sued in a partial comparative negligence jurisdiction. The jury found her 60% responsible and the bookstore owner 40% responsible, so she can recover 40% of her damage. (D) Because she sued in a contributory negligence jurisdiction, and the jury found her more responsible than the bookstore owner.

The correct answer is: (B) Because she sued in a pure comparative negligence jurisdiction, and the jury found her 90% responsible and found the bookstore owner only 10% responsible. The second customer would be able to recover for the portion of her damages for which the bookstore owner was considered responsible (10%). Under a comparative negligence regime, where the plaintiff's negligence has contributed to her own injuries, the total damages caused by the defendant may be apportioned based upon a determination of the relative fault of each party. A majority of states have adopted comparative negligence systems, either by statute or judicial decision. Some states have adopted a "pure" system of comparative negligence. In such states, apportionment of damages tracks apportionment of fault perfectly--if the defendant is 25% responsible and the plaintiff is 75% responsible, the plaintiff recovers from the defendant 25% of the total damages she suffered.

After a long, drawn-out divorce from his wife, a husband was extremely angered at the judge's distribution of the marital assets. The wife had been awarded the house, both cars, the dogs, and custody of their two children. The husband decided that because he had lost everything in the divorce, it was only fair that his ex-wife should suffer just as much as he had. One night, when he expected his ex-wife and the children to be out, he entered her house with the intent of burning it to the ground. He had just begun pouring lighter fluid over the living room floor when he heard a noise from the kitchen. Startled, he fled the house, jumped into his car, and drove away as fast as he could. He was just about to turn into the parking lot of his own apartment building when a 10-year-old boy rode by on a bike. Distracted from the events of the evening, the husband did not notice the boy. His car struck the boy head-on, killing the boy instantly. Of what crime, if any, is the husband likely to be convicted with regard to the killing of the 10-year-old boy? (A) Involuntary manslaughter. (B) Felony murder. (C) First-degree murder. (D) No crime, because it was an accident.

The correct answer is: (B) Felony murder. When one causes the death of another during the commission of a dangerous felony or the immediate flight therefrom, he is guilty of felony murder. The felony is deemed to have terminated only when the defendant has reached a place of temporary safety. Here, the husband was fleeing from his attempted arson (an inherently dangerous felony) at the time that his car struck the boy and had not yet reached a place of temporary safety. As such, the husband is guilty of felony murder.

A yoga teacher and her pupil were on their way to school in the pupil's car when a police officer pulled them over for not stopping at a stop sign. The pupil was driving, and the police officer approached the vehicle to ask the pupil for his driver's license. As the officer approached the vehicle, he noticed the butt of a handgun poking out from the yoga teacher's shirt. The officer immediately ordered the yoga teacher and the pupil to exit the vehicle, and he frisked them. At that time, the yoga teacher told the officer that she was a convicted felon illegally in possession of a firearm, and she voluntarily handed over the handgun to the officer. The police officer did not discover any weapons or contraband on the yoga teacher's pupil. The police officer issued the pupil a ticket, and arrested the yoga teacher for illegal possession of the handgun. Before releasing the pupil, and without asking for consent, the police officer conducted a search of the interior of the pupil's car. In the glove compartment, the officer found an ounce of marijuana. The officer then arrested the pupil for possession of illegal drugs. At the pupil's criminal trial, the pupil's attorney moved to exclude the evidence of the marijuana found in the pupil's vehicle. Should the court exclude the evidence? (A) No, because the police officer reasonably believed that there was contraband in the car. (B) No, because the search of the pupil's car was an appropriate search incident to arrest. (C) Yes, because the pupil's car was not lawfully impounded. (D) Yes, because the pupil was pulled over for a traffic violation, not on suspicion of possession of illegal drugs.

The correct answer is: (B) No, because the search of the pupil's car was an appropriate search incident to arrest. Police officers may conduct a search incident to arrest which includes the passenger compartment of a vehicle if it is reasonable to believe that the person arrested might be able to access the vehicle at the time of the search or that the vehicle may contain additional evidence of the offense which caused that person to be arrested. When a police officer issues a traffic citation which does not result in a custodial arrest, the officer may not search the driver or the car for any contraband. However, when a person in a vehicle is arrested, the officer has the authority to search the interior of the vehicle, even when that officer does not fear for his safety or believe that any contraband will be found. That search must be limited to the immediate control and wingspan area of the person arrested. In this case, although the pupil was initially detained on the basis of a traffic violation, the yoga teacher, who was riding in the vehicle, was arrested for illegal possession of a firearm. As such, the police officer had the right to search the interior of the vehicle, and the marijuana discovered during this search is therefore admissible at the pupil's criminal trial.

An investor purchased a 100-acre piece of land 30 years ago. Ten years later, the investor built a house on a different residential property and moved in. The investor instructed an accountant to pay all property taxes on both the 100-acre piece of land and the residence, and to inspect the 100-acre piece of land once every five years. Shortly thereafter, a trespasser cleared a five-acre section in the northwest corner of the 100-acre piece of land. The trespasser built a house, a barn, and a store and posted a sign, "Welcome to my town." The trespasser never entered the other 95 acres of the land. Twenty years after the trespasser entered the property, the investor sold the 100 acres to a developer. The developer announced plans to build a mall and proceeded to clear the remainder of the property. The trespasser sought an injunction to stop the developer. The relevant statute of limitations is 10 years. Will the trespasser succeed in proving that the trespasser has acquired title to the whole of the 100-acre property? (A) No, because the property was inspected every five years. (B) No, because the trespasser acquired title to only the five acres of land that he had cleared and used. (C) Yes, because the trespasser obtained title to all of the 100-acre property by adverse possession. (D) Yes, because the developer should have known the trespasser was in possession of part of the property.

The correct answer is: (B) No, because the trespasser acquired title to only the five acres of land that he had cleared and used. Under the doctrine of adverse possession, the occupation of only a portion of the property will confer title to the whole only if the claimant enters under color of title (constructive adverse possession). In this case, the trespasser did not enter under color of title. However, the trespasser will still be able to claim possession of the five-acre portion of the property that he inhabited and used, because his possession was open, visible, and notorious; actual; exclusive; hostile and under a claim of right; and continuous for the required statutory period.

An actress had a lease for a loft in a busy city neighborhood. Her boyfriend frequently visited but never spent the night at the apartment with her and he was not on the lease. The boyfriend got into a fight in front of the loft and the police were called by a witness that saw the boyfriend run into the actress's apartment. When police arrived at the loft they knock and the actress answered the door. Police asked if they could enter and search the apartment and the actress gave officers permission to enter and search. The officers located numerous stolen and counterfeit checks made payable to the boyfriend, a bag of cocaine, and the boyfriend's government identification card on the kitchen counter. The boyfriend is arrested for the all of the illegal items that the police found. At trial, the boyfriend's attorney argues that the exclusionary rule should apply and the evidence should be excluded. Does this rule apply? (A) No, this rule is only available in deportation and parole revocation hearings. (B) No, the boyfriend does not have standing to raise a Fourth Amendment violation claim. (C) Yes, it is a mandatory remedy for a violation of the defendant's Fourth Amendment rights. (D) Yes, the police did not have a warrant to search and seize.

The correct answer is: (B) No, the boyfriend does not have standing to raise a Fourth Amendment violation claim. The exclusionary rule is used to deter unlawful police conduct by making inadmissible any evidence or statements that were obtained in violation of the Fourth Amendment. However, before a defendant can assert a Fourth Amendment violation claim, that defendant must have standing and must personally be the victim of the police's unreasonable conduct. A court is tasked with determining if the defendant has a legitimate expectation of privacy in the invaded place. This apartment belonged to the actress and not the boyfriend. The boyfriend was not listed on the lease and was never more than an overnight guest. The actress, the lawful occupant of the property, gave police permission to search the location and they located the illegal items in plain view. The boyfriend does not have standing the challenge the search and seizure and has no legitimate expectation of privacy in this space. Additionally, there was nothing improper about the police entry into the apartment since they were given permission to enter by the actress.

A police helicopter was searching for a fleeing suspect who had just robbed a convenience store when the officers in that helicopter observed what they thought was growing marijuana plants in the yard of one of the homes in the neighborhood. A tall gate surrounded the home's backyard making it impossible to see inside of the backyard from the ground. The police reported their findings to a detective. The detective called the homeowner and requested consent to search the backyard. The homeowner told the officers that she was currently out of the country, had been for the past six months, and that her friend was house sitting while she was gone. The homeowner then told the detective that the police could search the house. That friend was not in the home at the time the police arrived to search. When police searched the backyard, they found a large quantity of marijuana plants and charged the housesitting friend with growing, manufacturing, and trafficking in marijuana. Can the housesitting friend have the marijuana suppressed before trial? (A) No, the house-sitting friend does not have standing the challenge the search. (B) No, the housesitting friend had no reasonable expectation of privacy and the homeowner consent was valid. (C) Yes, the police officers conducted an aerial search without a warrant. (D) Yes, the homeowner consent was invalid.

The correct answer is: (B) No, the housesitting friend had no reasonable expectation of privacy and the homeowner consent was valid. In this fact pattern, there are two different means by which the police observe the property. First they observe from overhead in a helicopter. They then observe the marijuana by walking through the backyard with the homeowner's consent. As to the aerial observation, the Supreme Court has ruled that there is no reasonable expectation of privacy from naked-eye observations of property from the air. The observation is therefore technically not a search within the meaning of the Fourth Amendment and so no warrant was required. There is, however, a reasonable expectation of privacy from police entering a private, fenced-in backyard that is not visible from the street. The subsequent walk-through of the property therefore does constitute a search. With this established, the analysis shifts to whether the warrantless search was subject to one of the enumerated exceptions. In this case, the consent exception is applicable. Even third-party consents may be valid against the accused where the person granting such consent has either actual or apparent authority to consent to the search. While a landlord cannot consent to the search of a tenant's apartment, nothing in the fact pattern here suggests that the homeowner (as opposed to a landlord) relinquished the homeowner's possessory interest in the property. On the facts here, the homeowner probably had actual authority to consent to the search, or, at the very least, apparent authority, and in either case the consent to the search is valid.

A ten-year-old boy lived with his mother in a housing project. Every day, he walked to a private charter school, which was about ten blocks from his home. In response to complaints from many parents of school-aged children, the school had hired crossing guards for the busiest intersections near the boy's school. The guards were on duty from 7:30 a.m., when students arrived, until 3:15 p.m., by which time most of the school's students had returned home. Because the boy's mother worked, the boy participated in an after-school program which allowed him to remain under adult supervision for most of the afternoon. About ten percent of the students in the boy's school participated in the program. Following the after- school program, the boy walked home from school at about 4:30 p.m. each day and had to cross streets unaided. The school was aware of the absence of guards late in the day, but did not request that the city station guards for the late-departing students. One day while the boy was walking home, he entered an intersection that was guarded during school hours. While in the crosswalk, he was struck and injured by a vehicle operated by a driver. The driver admitted that he was driving too fast and did not see the boy in the crosswalk. The boy's mother files suit on the boy's behalf against both the driver and the school. Will the boy's mother prevail against the school? (A) No, because the driver's negligence caused the accident. (B) No, unless the school owed a duty to the boy. (C) Yes, unless the boy's mother was negligent. (D) Yes, because the boy was a business invitee.

The correct answer is: (B) No, unless the school owed a duty to the boy. The plaintiff in a negligence action must show that the defendant has a legal duty to protect the plaintiff against an unreasonable risk of harm and that the defendant failed to conform his conduct to the legally required standard. If the school did not owe a duty to the boy to keep him safe off the premises, the boy's mother cannot prevail in her lawsuit against the school.

The defendant considers himself the greatest and most inventive practical joker on earth. He plans a dinner party incorporating an elaborate joke to spook his guests. The planned joke involves a timed fire near the dining room door, set to go off as the defendant's guests are eating dinner. The defendant has treated the walls, door, ceiling, and floor of the room with a special fireproof finish to prevent them from igniting, and the fire is set to burn for only 45 seconds. On the night of the dinner party, the defendant is electric with anticipation; he cannot wait to see the terrified looks on his friends' faces. However, when the fire goes off as planned, a woman, one of the guests, panics and runs from the room through the only doorway, which is precisely where the flames are. As the woman runs through the flames, her clothes catch fire, and she dies from her injuries. The defendant should be convicted of which of the following crimes? (A) First-degree murder. (B) Second-degree murder. (C) Voluntary manslaughter. (D) No crime.

The correct answer is: (B) Second-degree murder. In most states, murder is divided into two degrees for the purpose of imposing a more severe penalty for some murders than for others. First-degree murder includes intent-to-kill murder accompanied by premeditation, deliberation, and murder in the commission of any of several named serious or inherently dangerous felonies ("BARRK"--burglary, arson, rape, robbery, kidnapping). Murder not falling within any of the first-degree murder categories is considered second-degree murder.

While doing sit-ups at the gym, a man was struck in the head by a large weight plate that the woman next to him accidentally dropped. The woman was a new member of the gym, and had not taken the informational class the gym offered to new gym members. The man was knocked unconscious and suffered a concussion. An ambulance rushed to take the man to the hospital. On the way, it suddenly began to storm very heavily. The ambulance driver lost control of the ambulance, and it crashed into a telephone pole. The man, still unconscious, was knocked into some equipment in the ambulance, and suffered a broken leg. What, if anything, is the woman liable for? (A) The broken leg. (B) The concussion. (C) The broken leg and the concussion. (D) Nothing.

The correct answer is: (B) The concussion. This question is testing causation. A plaintiff must prove the element of causation to prevail on a negligence claim. Proximate cause will be found if the defendant's conduct is a proximate cause of the injury. The test for proximate cause is whether the defendant should have reasonably foreseen the general consequences of the type of harm the plaintiff suffered. Defendants are not liable for superseding causes. A superseding cause breaks the chain of causation between the initial wrongful act and the ultimate injury. An example of a superseding cause is an act of God. In this situation, while the woman is definitely liable for any direct injuries from hitting the man with the weight plate, she will not be liable for the broken leg as it occurred because of the storm.

A convenience store sold specialty frozen drinks in a variety of flavors, including sugar-free. One weekend, a new employee mistakenly filled the sugar-free frozen drink machine with the regular sugary frozen drink mix. A diabetic surgeon came in to purchase his standing favorite, a large raspberry sugar- free frozen drink. Due to the employee's mistake, the surgeon consumed far more sugar than his system could tolerate. Within an hour of drinking the sugary drink, he was feeling woozy and lapsed into a diabetic coma shortly thereafter. He was admitted to the hospital and successfully treated, but he continued to experience blurriness of vision, which doctors were hopeful, but not certain, would improve in time. A world-renowned trumpet player who was suffering from carpal tunnel syndrome had been scheduled for surgery with the surgeon, whose specialty was operations to address carpal tunnel syndrome. However, given the surgeon's medical problems, the surgeon was unavailable to perform the surgery as scheduled, and the trumpet player found another surgeon to perform the surgery. However, the surgery failed to improve the trumpet player's condition, and following the surgery, he had lost all mobility in his wrist and was unable to play the trumpet. A production company had hired the trumpet player to play trumpet on an upcoming movie soundtrack, pending his surgery. Based on the trumpet player's famous name, the production company had lined up numerous large investors in the film. When the trumpet player came out of the surgery unable to perform on the soundtrack as planned, the company's investors backed out of the deal, causing the company major financial losses. The company filed a claim against the convenience store seeking recovery of its severe financial losses. The jurisdiction applies a comparative negligence standard. Which of the following is the strongest argument in defense of the convenience store? (A) The convenience store is not liable for the company's damages, because the company was not a customer of the store. (B) The convenience store is not liable for the company's damages, because the company's damages were too far removed from the store's negligent act. (C) The convenience store is not liable for the company's damages, because the gross negligence of the trumpet player's surgeon constitutes a superseding cause of the company's damages. (D) The convenience store is not liable for the company's damages, because the company could have found a replacement trumpet player.

The correct answer is: (B) The convenience store is not liable for the company's damages, because the company's damages were too far removed from the store's negligent act. A defendant's duty of care is owed only to foreseeable plaintiffs i.e., those within the risk of harm created by defendant's unreasonable conduct. In the same vein, the doctrine of proximate cause acts as a limitation on liability based on the extent to which the risk created by the defendant's unreasonable conduct will allow recovery for the harm suffered by the plaintiff. The Restatement 2d approach allows a defendant's liability to be cut off where the court finds highly extraordinary the relationship between the defendant's conduct and the resulting harm. Alternatively, many jurisdictions extend liability only to the foreseeable consequences of defendant's negligence. In this instance, there exist a number of factual uncertainties, including whether the trumpet player's surgery would necessarily have succeeded if performed by the surgeon and the uncertainty of financial investors in general. While it is possible that a court might still find that the damages suffered by the company were sufficiently linked to the negligence of the convenience store as to make the store liable, the store's strongest defense lies in its assertion that the plaintiff and the damages suffered were so unforeseeable as to make it inappropriate for the convenience store to bear liability for the company's injury. As such, this is the best response.

A kayaker was eager to get his craft in the water after a long winter. There had been significant snowfall over the winter, a very late thaw, and heavy rains in April. That meant that the water was high and moving very fast in the river where the kayaker liked to paddle. Aware of the danger, but tired of waiting for the water to recede, the kayaker put his craft in the river one warm May day and started to paddle downstream. He had traveled only a few hundred yards when his kayak was swamped, he lost his oar, and he went careening toward a section of dangerous rapids with no way to control the kayak. As he neared a bridge, a woman fishing off the bridge saw that the kayaker was floating without an oar. She tied herself to the bridge and dove into the river in an attempt to grab the kayak. The fisher successfully rescued the kayaker, but in the process suffered some injuries herself, for which she required hospitalization. The fisher sued the kayaker for the injuries she sustained in the rescue. Who will prevail? (A) The fisher, because she saved the kayaker's life. (B) The fisher, because it was foreseeable that someone would try to rescue the kayaker. (C) The kayaker, because the fisher assumed the risk of injury. (D) The kayaker, because he did not place the fisher in danger.

The correct answer is: (B) The fisher, because it was foreseeable that someone would try to rescue the kayaker. A plaintiff seeking to establish liability in a negligence action must prove that the defendant owed a legal duty to the plaintiff, that the defendant's conduct breached that duty, that the defendant's breach of duty was the actual and legal (proximate) cause of injury to the plaintiff, and that the plaintiff suffered damages as a result of the defendant's conduct. The element of "duty" is usually described as an obligation, recognized by law,that requires the defendant to conform to a certain standard of conduct for the protection of others against unreasonable risk of harm. Under the "rescue doctrine," a tortfeasor who creates a dangerous situation owes an independent duty of care to the rescuer, which arises even when the defendant endangers no one's safety but his own. The original wrongdoer remains liable for the harm he causes, regardless of whether the rescuer succeeds and whether the rescuer injures himself, the person rescued, or a stranger. Here, it was reckless for the kayaker to enter the water that was still swollen due to the spring thaw, and it was foreseeable that someone would try to rescue the kayaker once he lost control of his craft. The kayaker is liable to the fisher for her injuries.

A boy was a 6th grade student in a junior high school. The boy was a particularly difficult student and was frequently involved in verbal and physical fights in school. The boy's teacher was usually the one to break up fights begun by the boy. On one particular day, the teacher observed the boy walking toward the stairwell that the teacher knew was in disrepair. One of the steps near the top of the stairwell was missing. Just as the teacher was about to call out to warn the boy, the teacher recalled the broken finger she had suffered during the last fight she broke up involving the boy. Instead of warning the boy, the teacher decided to stand back and watch the boy get what was coming to him. The boy tumbled down the stairs and was rendered unconscious. Soon after, the boy woke up in the infirmary. The boy sued the teacher for negligence. Which of the following is correct? (A) The teacher will not be held liable for negligence, because she had been injured helping the boy in the past. (B) The teacher should be held liable for negligence. (C) The teacher will not be held liable for negligence, because the teacher did not cause the boy's injury; the fall down the stairs did. (D) The teacher will not be held liable for negligence, because the teacher was not responsible for keeping the stairwell in good repair.

The correct answer is: (B) The teacher should be held liable for negligence. The teacher's teacher/student relationship with the boy gives rise to an affirmative duty to protect the boy from injury. This special relationship between defendant and plaintiff takes place when the defendant derives economic benefit or when the defendant occupies a position of power over the plaintiff. The following relationships have been generally recognized as triggering the duty to care for the plaintiff: (1) employer/employee (during and in the scope of employment); (2) common carrier and innkeeper/customer; (3) school/pupil; (4) parent/child; and (5) jailer/prisoner. Consequently, the teacher breached her duty to the boy when she failed to warn the boy of the dangerous stairs.

An instructor hired by a corporation took several of the corporation's workers on a hike into some foothills to teach them some lessons on how to trust one another and work together better as a team. During one of the exercises, he demonstrated a "trust fall" in which one of the workers would let himself fall backwards while trusting that the rest of the workers would catch his fall. This worked out well, but when the last worker got up to try, the instructor became frightened by a bee and negligently ran away in the middle of the trust fall before he could give the signal to the other workers to catch the worker. The worker suffered physical injuries to his tailbone and foot as a result of the botched fall. Assuming the instructor is liable in negligence for the injuries suffered from the fall, for which of the following subsequent injuries would the instructor least likely be liable? (A) The worker's fellow workers negligently transported the worker out of the foothills, causing him the loss of two of his fingers. (B) The worker cried in pain at such an annoying frequency that another worker punched him in the face to quiet him down, causing injuries to his face. (C) The doctor who treated the worker accidentally operated on the wrong foot, causing injuries to that foot. (D) The worker had to remain in bed for two months after the injury and developed muscular injuries as a result.

The correct answer is: (B) The worker cried in pain at such an annoying frequency that another worker punched him in the face to quiet him down, causing injuries to his face. The original tortfeasor is generally liable for subsequent injuries suffered by a victim which are a foreseeable consequence of the original injury. This does not include intentional torts. Therefore, because the other worker committed an intentional battery against the worker, the instructor will not be liable for the worker's facial injuries.

A juggler borrows money from his coworker. The juggler is unable to pay the money back, and years pass. The coworker eventually needs the money and contacts the juggler, who consults a lawyer. The lawyer tells the juggler that the statute of limitations on the debt has expired and that he is not legally obligated to repay the loan. The juggler nevertheless promises to do so in writing, but then changes his mind. May the coworker enforce the juggler's recent promise to repay the old loan?(A) Yes, because the past consideration is sufficient to support a present promise. (B) Yes, because it was a promise to repay a debt that, but for the statute of limitations, would still be owing. (C) No, because the recent promise to repay the money was a gratuitous promise. (D) No, because the recent promise to repay the money was supported only by past consideration.

The correct answer is: (B) Yes, because it was a promise to repay a debt that, but for the statute of limitations, would still be owing. This answer is correct because it applies an exception to the general rule that past consideration will not support a contract. A written promise to pay a debt that is barred by the statute of limitations is one of two particular types of promises that are enforceable even if there is no new consideration for them. In this case, the juggler's promise to pay the debt despite the expiration of the statute of limitations is therefore binding. [Note: This questions requires you to know the exceptions to the rule for past consideration. Most students, not knowing this exception, will select "D".]

The head chef of a prestigious cooking school sponsored a contest. He had all instructors post the following notice: "This cooking institution hereby offers a one-year scholarship for the coming academic year to any student currently enrolled in this institute who enters and wins this year's Healthy Chef competition. This is the annual competition sponsored by Healthy Life magazine. All menus and recipe entries must be submitted to the Head Chef's office by January 10, as well as to Healthy Life magazine." A cooking student saw the notice and decided to enter the contest. He left a note on the main counter of the Head Chef's office which stated, "I accept the Head Chef's offer of a one-year scholarship upon winning this year's Healthy Chef competition." On January 1, the student sent his original menu and recipes to Healthy Life magazine for entry in the competition. He simultaneously submitted copies to the Head Chef's office. On April 15, Healthy Life magazine notified the student that he had won the Healthy Chef contest. The student was delighted, and in reliance upon the promised one-year scholarship, he did not submit his tuition payment for the following year. When he was contacted by the school's bursar's office regarding the unpaid tuition, he told the administrator that his tuition was covered by a one-year scholarship. The administrator informed the student that if he did not submit tuition payment within 15 days, he would be expelled. Is the Head Chef's one-year scholarship offer enforceable? (A) Yes, because the school's unilateral contract became a binding bilateral contract when the student notified the dean's office of his intent to enter the competition. (B) Yes, because the cooking school's notice of conditions to enter the cooking contest constituted a unilateral contract, which the student accepted by entering and winning the contest. (C) No, because the scholarship offer was contingent upon the fulfillment of a condition involving Healthy Life magazine, an outside party. (D) No, because it was not certain that the student would win the contest.

The correct answer is: (B) Yes, because the cooking school's notice of conditions to enter the cooking contest constituted a unilateral contract, which the student accepted by entering and winning the contest. Typically, an advertisement of an offer is considered an invitation for offers, rather than an offer itself. However, where an offeror requests the performance of an act instead of a promise to perform, as in the case of reward offers, a unilateral contract is formed. A unilateral contract may be accepted only by the offeree's actual performance; a promise to perform is not sufficient to create acceptance. Here, the student's acceptance of the scholarship offer came when he submitted his menu and recipes entry in the Healthy Chef competition and won the contest. Upon his winning of the contest, the unilateral contract became an enforceable bilateral contract. The student is therefore entitled to the school's full performance under that contract, and hence the one-year scholarship is enforceable.

A burglar, carrying a realistic looking toy gun, entered a home. The burglar told the homeowner to give the burglar all the wife's jewels or the burglar would shoot. The homeowner then removed a pistol from his pants and shot the burglar dead. The homeowner was charged with murder. The homeowner raised a defense of self-defense. Was the defense justified? (A) Yes, because a homeowner is always allowed to defend using deadly force. (B) Yes, because the homeowner's force was reasonably necessary. (C) No, because the homeowner was the initial aggressor. (D) No, because the homeowner did not retreat.

The correct answer is: (B) Yes, because the homeowner's force was reasonably necessary.Yes, the homeowner's force was reasonably necessary to justify self-defense. A person may use that amount of force in self-defense that is reasonably necessary to prevent imminent bodily harm. Here, the homeowner had a reasonable belief that deadly force was necessary because the burglar had a realistic looking gun and said that he would "shoot". The fact that the gun is a toy would not negate the defense because the homeowner's belief that the toy gun was real was reasonable. Test tip: In a criminal question, focus on picking the most important fact that helps you decide whether the defendant is guilty or not. Here, it is the victim's threat of using a toy that does look like a real gun, which justifies the use of self-defense.

A custodian owned a 500-acre parcel of land. A mechanic lived on a parcel bordering the north side of the parcel. Realizing that the mechanic lacked convenient access to the new highway and shopping center located along the south side of the parcel, the custodian granted an easement to the mechanic so that the mechanic could walk across the parcel to reach the highway. The mechanic promptly recorded the easement. Nine years later, the mechanic opened a used car dealership on his lot and gave his customers permission to drive along the easement across the parcel. The farmer became annoyed at the steady stream of cars that were now crossing over his land. One year later, when the mechanic refused to tell his customers to stop using the easement, the custodian filed for an injunction to stop the mechanic's customers from driving across his land. The statutory period for adverse possession is 10 years. Is the court likely to grant the injunction? (A) Yes, because the grantor of an easement appurtenant retains the right to extinguish the easement. (B) Yes, because the mechanic's use of the easement exceeded the legal scope of the easement. (C) No, because the mechanic's customers had acquired an easement by prescription. (D) No, because the custodian created an easement in gross.

The correct answer is: (B) Yes, because the mechanic's use of the easement exceeded the legal scope of the easement. Excessive use of an easement does not forfeit or extinguish the easement. In such cases, the servient tenant may enjoin the dominant tenant's excessive use of the easement, but the dominant tenant may still use the easement within the scope for which it was created. In this case, the custodian can enjoin the used car lot customers from using the easement but the mechanic could still use the easement as a walkway across the custodian's land.

A man loses his driver's license as a mandatory penalty for a simple possession of marijuana under state law. Under the statute, even if the possession occurs nowhere near a car, as was the case with this man's violation, the suspension still applies. The man has never committed any sort of traffic violation, been issued a speeding ticket, or been involved in an automobile accident in his 15 years of driving. In the jurisdiction, driving without a license is a misdemeanor offense. Several weeks following the revocation of the man's license, his wife comes down with the flu and cannot drive him to work as usual. Because he needs to get to work, the man drives for the first time since his license was suspended. He is less than a mile away from home when a child darts out from between two cars and into the street, directly into the path of the man's car. The child is killed instantly in the collision. There is no indication that at the time of the accident the man was operating his motor vehicle less than safely or committing any moving violation. If the man is charged with involuntary manslaughter, he will likely be found: (A) not guilty, because he did not intend to kill the child. (B) not guilty, because the misdemeanor that was committed bore no proximate relation to the death. (C) not guilty, because he was not speeding at the time of the accident. (D) guilty, under the misdemeanor manslaughter rule.

The correct answer is: (B) not guilty, because the misdemeanor that was committed bore no proximate relation to the death. The misdemeanor manslaughter rule applies when the underlying misdemeanor is a malum in se (inherently wrong) offense. Here, the man's only offense was driving without a license. Because the act of driving without a valid license is not inherently wrong, but is "wrong" simply because the legislature so decreed, it is a malum prohibitum offense, not a malum in se offense. Death resulting from a malum prohibitum crime is sufficient to impose liability for involuntary manslaughter only when the killing is either a foreseeable consequence of the unlawful conduct or amounts to criminal negligence. Here, inasmuch as the death of the child was not a foreseeable consequence of the man's act of driving without a valid license, he will be found not guilty of misdemeanor manslaughter.

The State of Madison has enacted a statute providing that (1) first degree murder is "a deliberate and premeditated killing"; (2) second degree murder is "an unlawful killing with malice aforethought"; and (3) manslaughter is "either an unlawful killing committed with adequate provocation or an unlawful killing committed through criminal negligence." A worker, a resident of the State of Madison, is distraught over his recent dismissal from his job, which he had held for 20 years. The day following his dismissal, the worker returns to his former employer's place of business with two loaded handguns. The worker enters the building and confronts the owner of the business. Waving the two handguns around indiscriminately, the worker shouts, "You'd better give me my job back, or I'm going to start shooting!" Concerned for the safety of her employees and customers, the owner dives at the worker in an attempt to disarm him. As the owner tackles the worker, one of the handguns hits the ground and discharges. A customer is shot and killed. The worker is guilty of: (A) First-degree murder. (B) Second-degree murder. (C) Manslaughter. (D) No crime.

The correct answer is: (B) second-degree murder. The State of Madison defines second degree murder as "an unlawful killing with malice aforethought." "Malice aforethought" includes the intent to kill, the intent to inflict serious bodily injury, and reckless indifference toward a substantial and unjustifiably high risk of death. At a minimum, the worker's actions show a reckless indifference toward a high risk of death. The worker is guilty of second degree murder.

A police officer witnessed a robber point a gun at a woman and take her watch. The officer chased the robber on foot. The robber, with gun still in hand, ran into a nearby house. The officer followed the robber into the house and began to search the house in an attempt to locate the robber. The officer went into the first bedroom and looked under a bed and found a large quantity of cocaine. He continued his search and found the robber in the bathroom and arrested him. During the discovery phase the prosecutor determined that the robber owned the house. The prosecutor charged the robber with armed robbery and possession of drugs with the intent to distribute. At a pre-trial motion, the robber's attorney argues that the drugs are inadmissible because the officer did not have a warrant to search the house. How will the court rule? (A) Inadmissible, the officer needed a warrant to enter the house. (B) Inadmissible, the drugs were not in plain view. (C) Admissible, the officer was in hot pursuit. (D) Admissible, there were exigent circumstances.

The correct answer is: (C) Admissible, the officer was in hot pursuit. Police may enter and search a private dwelling while in reasonable pursuit of a fleeing suspect. In such a circumstance, what would otherwise have been a warrantless search becomes lawful because the police are in actual "hot pursuit" of a fleeing suspect and actively attempting to apprehend him. While in that pursuit, they may seize not only evidence of the crime for which they are chasing the suspect but also any additional contraband that they find. Here, the officer was in hot pursuit of the robber and while attempting to apprehend him, the officer chased the robber into a house. The drugs were located under a bed while the officer was looking for the robber. Therefore, the drugs were lawfully seized contraband and can be admitted as evidence.

An avid collector's collection of Union and Confederate mortar shells was the world's largest collection of Civil War memorabilia. Feeling that he was obligated to share his piece of American history with the nation, the collector regularly loaned his collection to museums and historical societies for exhibits. The historical society of the collector's hometown borrowed the collection for an exhibit celebrating the contributions the city made to the country and the world. Volunteers from the community offered their assistance to the historical society, a not-for-profit organization, in setting up the exhibit and transporting the mortar shells from the collector's house to the exhibit hall. One of the volunteers used his pickup truck to move some of the shells. On the road from the collector's home to the hall, the tailgate of the volunteer's truck opened, spilling mortar shells onto the road. None of the shells were damaged, but one of them struck a parked car causing extensive damage to the vehicle. The transportation of the mortar shells was televised live on the local news, and the car's owner, who was watching the news, witnessed the near destruction of his car, a rare and antique automobile. If the collector brings a negligence action against the volunteer for allowing the mortar shells to spill onto the road, what will he be entitled to recover? (A) Punitive damages. (B) Nominal damages. (C) Neither nominal nor punitive damages. (D) Both nominal and punitive damages.

The correct answer is: (C) Neither nominal nor punitive damages. The elements of a negligence action are a duty of care, a breach of that duty, actual and proximate causation, and damages. In seeking to establish liability in negligence, the collector must prove that: the volunteer owed a legal duty to the collector; the volunteer's conduct breached that duty; the volunteer's breach of the duty was the actual and legal (proximate) cause of injury to the collector; and the collector suffered damages as a result of the volunteer's conduct. Here, given that the mortar shells were not harmed in the accident, the collector did not suffer any damages. As such, the collector is not entitled to recover.

A plaintiff sued a defendant for violation of the Federal Age Discrimination in Employment Act, which created the cause of action that the plaintiff asserted. The parties agreed to settle the case and entered into a contract entitled "Settlement Agreement and Mutual Release," in which the plaintiff agreed to release all claims against the defendant in exchange for a future payment of $80,000. The plaintiff and the defendant were both citizens of State A. The defendant failed to make the $80,000 payment when it was due, and the plaintiff filed suit for breach of the settlement agreement in federal court. The defendant moved to dismiss the case for lack of federal subject matter jurisdiction. The court denied the motion. Was the court correct to deny the motion to dismiss for lack of federal subject matter jurisdiction? (A) Yes, because more than $75,000 was in controversy. (B) Yes, because the plaintiff's settled claim under the Age Discrimination in Employment Act arose under federal law. (C) No, because the case for breach of the settlement agreement did not arise under federal law. (D) No, because the plaintiff's settled claim under the Age Discrimination in Employment Act should not have been in federal court to begin with.

The correct answer is: (C) No, because the case for breach of the settlement agreement did not arise under federal law. The plaintiff had a claim that arose under federal law but released it as part of the settlement. The plaintiff's claim now arises solely from the breach of contract, which is a state law claim.

Police officer stopped a vehicle with expired license plate registration. The officer asked the driver for his license and observed a hand gun on the front seat next to the driver. The officer asked for gun registration and the driver admitted it was not a registered weapon. The officer ordered the driver out of the car and arrested him for possession of an unregistered firearm. The officer then ordered the passenger to exit the car. As the officer was conducting a pat down search of the passenger, a bag of cocaine fell out the passenger's shirt pocket. The officer then arrested the passenger on drug possession. The passenger has filed a motion to suppress the evidence of the cocaine claiming that the officer did not have probable cause to pat him down after the driver was handcuffed for the illegal gun possession. Should the court grant the motion to suppress? (A) Yes, because the driver admitted the gun belonged to him and not the passenger. (B) Yes, because the passenger was an innocent bystander. (C) No, because the officer had a reasonable fear for his own personal safety and the drugs fell to the ground in plain view. (D) No, because the passenger was an accomplice to the driver.

The correct answer is: (C) No, because the officer had a reasonable fear for his own personal safety and the drugs fell to the ground in plain view. A police officer has the right to order the driver to exit the vehicle during a routine traffic stop [Pennsylvania v Mimms]. This rule was extended to allow police to order the passenger out of the vehicle during a routine traffic stop [Maryland v Wilson]. The opinions recognize that officers have the right to ensure their own safety during routine traffic stops. The pat down search was likewise permissible once the officer determined that the gun was not registered. The cocaine fell out of the passenger's pocket and was observed in plain view prior to the seizure. The initial stop and detention were lawful and, therefore, the subsequent pat down and drug seizure were also lawful.

After the federal government issued an orange terrorist alert in the county, the county board set up road blocks authorizing state troopers and homeland security officers to board passenger buses and request passengers to produce identification, identify baggage, and agree to a baggage search. A bus destined for Florida was boarded by a homeland security officer who announced that passengers were free to leave the bus at any time and that any passenger had the right to refuse a search of that passenger's baggage. The officer approached an elderly passenger who produced his identification and pointed out his bag. The officer then asked the passenger if the officer could search the bag and the passenger shrugged his shoulders and sat down. When the officer looked inside the bag he discovered two bricks of marijuana. The passenger was arrested, Mirandized and taken into custody. In a pre-trial motion, the defendant moved to suppress the evidence of marijuana. Should the court grant the motion? (A) Yes, because the warrantless search violated the Fourth Amendment. (B) Yes, because there were no exigent circumstances when the road blocks were authorized by the county board. (C) No, because the passenger consented to the search. (D) No, because body language is assertive conduct and is admissible as an admission into evidence.

The correct answer is: (C) No, because the passenger consented to the search. The issue here is whether the person of the defendant was illegally seized and thus making the subsequent seizure of the drugs the fruit of the initial unlawful intrusion (the fruit of the poisonous tree doctrine). The Supreme Court has held that a totality of the circumstances test should be used to determine whether the seizure of the person occurred; that is, whether a reasonable person would believe that he was free to leave, and thus, not in custody, and whether a reasonable person would feel free to decline the officer's requests and terminate the encounter. The Court observed that an officer boarding a bus does not automatically constitute a seizure of the person under the objective totality of the circumstances standard if a reasonable person would feel free to leave. Here, the officer expressly advised passengers that they were free to leave and that they had the right to refuse a baggage search. The officers did not block the bus exit. The elderly passenger was not subject to custodial interrogation, and Miranda warnings were not required. While the baggage search was a warrantless search, which is presumptively invalid, a warrant exception applied. The elderly passenger knowingly and intentionally gave his consent to the search of his property.

A dealer inherited a soybean plantation on her 21st birthday. The dealer relied on her business manager to manage the soybean plantation's profitable operations. On the dealer's 25th birthday, the business manager, posing as the true owner, sold the soybean plantation to a rancher. The business manager signed the warranty deed and kept the purchase money. The rancher subsequently moved in and tore up much of the crop, replacing the soybeans with a golf course and a shopping mall. On her 47th birthday, the dealer, who had long since forgotten about the soybean plantation, married a grocer. The dealer died one year later, leaving a will naming the grocer as her sole heir. The grocer soon discovered that the dealer had not received income from the soybean plantation for 23 years and went to inspect the soybean plantation. When the grocer learned of the soybean plantation's increased value, he ordered the rancher to vacate. When the rancher refused, the grocer brought an ejection action. The period to acquire title by adverse possession is 20 years. Will the grocer prevail in the ejection action? (A) Yes, because the rancher has not held the property for 20 years since the grocer inherited the property. (B) Yes, because the rancher's claim is based on title received by means of a fraudulent conveyance. (C) No, because the rancher has met the requirements for obtaining title by adverse possession.(D) No, because the rancher is a bona fide purchaser.

The correct answer is: (C) No, because the rancher has met the requirements for obtaining title by adverse possession. In order to obtain property by means of adverse possession, one's possession must be: (a) open, visible, and notorious; (b) actual; (c) exclusive; (d) hostile and under a claim of right or title; and (e) continuous for the entire statutory period. The rancher will prevail, because he met the requirements for obtaining title by adverse possession. For 23 years, the rancher continuously occupied the soybean plantation in an open manner not permitted by the dealer, and he held under the color of title from the sale by the business manager, so his claim will extend to the entire property described in the warranty deed, not just the parts he developed.

A developer purchased a lot in the downtown business district of Urbana on which a parking lot had been operated for years. The developer obtained the required city zoning approval and building permit to build an office building on the lot. The proposed office building would be twice as tall as an older apartment building on the adjacent lot. The owner of the apartment building believed that his tenants would be unhappy about the loss of light and obstruction of their view of a large city park on the other side of the developer's lot. The building owner brought an action to enjoin construction of the office building. Will the building owner succeed in obtaining injunctive relief? (A) Yes, because a subsequent use of property must not unreasonably interfere with the light and air of existing adjacent uses. (B) Yes, if the parking lot and any prior uses of the developer's property permitted light and air to reach the apartment building for the statutory period required for adverse possession. (C) No, unless the building owner has a cognizable property interest in light and air. (D) No, because the developer received the necessary city approvals and permits.

The correct answer is: (C) No, unless the building owner has a cognizable property interest in light and air. In order to succeed in enjoining the construction, the building owner must show a cognizable property interest. To do so, he would need to prove that he has an express easement for the light and air above the developer's current parking lot. However, easements for light and air must be created by express grant. Such easements will not be implied, even if the remaining elements for implication of a quasi- easement are present, because such easements are not apparent to purchasers of the servient estate. As such, this is the best response.

A woman was hit by a speeding car as she lawfully crossed the street. Aside from a broken leg, the woman seemed fine. The woman was taken to a local emergency room where she received a cast for her leg and went home. In the middle of the night, the woman woke up with a headache and severe vomiting. From the accident, the woman had suffered internal bleeding in her brain which caused long term brain damage. The doctor did not examine her head while she was at the hospital. What, if anything, is the driver of the car liable for? (A) The broken leg. (B) The injury to her brain. (C) The broken leg and the injury to her brain. (D) The driver of the car is not liable for anything.

The correct answer is: (C) The broken leg and the injury to her brain. In order to prevail on a negligence claim, a plaintiff will have to show both actual and proximate cause. This question is testing proximate causation. A defendant will be liable for the type of harm that is reasonably foreseeable. Further, a defendant will be liable for harm caused by foreseeable intervening causes. This includes ordinary negligence of the treating doctor. In this case, while the doctor may have been negligent in not picking up on the bleeding in her brain, there is nothing to indicate this was either intentional or grossly negligent. Thus, the driver of the car will be liable for both injuries.

A brother and sister inherited their childhood home from their father. They thereby became tenants in common. The property had never been used as anything except a residence. The brother had been residing on the property with his father at the time his father died. The sister had been residing in a distant city. After their father's funeral, the brother continued to live on the property. There was no discussion between the brother and the sister concerning their common ownership, nor had there ever been any administration of their father's estate. The brother paid all taxes, insurance, and other carrying charges on the property. He paid no rent or other compensation to his sister, nor did she request any such payment. Thirty years later, a series of disputes arose between the siblings for the first time concerning their respective rights. The jurisdiction where the land is located recognizes the usual common-law types of co-tenancies, and there is no applicable legislation on the subject. The jurisdiction has an ordinary 20-year adverse possession statute. The brother now claims fee simple title and brings an action against the sister to quiet title in himself. For whom should the court rule? (A) The brother, because he has exercised the type of occupancy ordinarily considered sufficient to satisfy the adverse possession requirements. (B) The brother, because the acts of the parties indicate that the sister acquiesced to the brother's right of ownership. (C) The sister, because there is no evidence that the brother has performed sufficient acts to constitute her ouster. (D) The sister, because one co-tenant cannot acquire title by adverse possession against another.

The correct answer is: (C) The sister, because there is no evidence that the son has performed sufficient acts to constitute her ouster. Under the unity of possession, each co-tenant is entitled to possess and enjoy the whole of the property subject to the equal right of his co-tenant. If one tenant wrongfully excludes another co-tenant from possession of the whole or any part of the whole property, such conduct amounts to an ouster. Here, choice (C) is correct because there is no evidence that the brother was wrongfully excluding the sister from the property. Choice (D) is incorrect because Burby points out that "possession may become adverse if the co-tenants are excluded from possession (i.e., ouster) or if the co-tenants are specifically notified of the adverse claim or can be charged with notice because of the acts of the possessor" (Real Property, p. 232).

A taxi driver was driving a passenger to the airport. The taxi driver, in a hurry, did not come to a full stop and rolled through a stop sign. An SUV struck the passenger side of the taxi. The passenger, who was four months pregnant, suffered a miscarriage from the stress of the accident; the passenger then fell into a depression. In a suit by the passenger against the taxi driver, can the passenger recover for the miscarriage and depression? (A) No, because as a guest in the taxi, the passenger can only recover if the taxi driver was reckless. (B) No, because the passenger's damages were not foreseeable injuries from the taxi driver's conduct. (C) Yes, because the taxi driver is liable for the unforeseeable extent of the passenger's injuries. (D) Yes, because the passenger was a foreseeable plaintiff of the taxi driver's negligence.

The correct answer is: (C) Yes, because the taxi driver is liable for the unforeseeable extent of the passenger's injuries. Where a defendant is negligent and brings about a plaintiff's injuries, the fact that the extent of the plaintiff's harm was unforeseeable does not prevent the defendant's liability. This is the "eggshell plaintiff rule," and the defendant is liable for the full extent of the plaintiff's injuries that resulted from the defendant's conduct. Here, the passenger's miscarriage and depression may not have been foreseeable injuries from the taxi driver's negligence, but the taxi driver is nonetheless liable for the full extent of the passenger's injuries.

A construction company is building a two-story plaza in town. The construction company has built a series of ramps that the carpenters and masons use to move materials from one level to another. These ramps remain set up even when the workers are not on-site. The teenage son of the site manager advises his father that many of his friends are using the ramps to jump into the air on their bicycles and skateboards. The next day, the site manager posts a sign at the site which states, "No Trespassing. Keep Off Ramps. Authorized Persons Only. No Skateboards or Bicycles." However, the site manager continued to find evidence that children had been riding bikes and skateboards on the property even after posting the signs. One week later, a 12-year-old boy rides his skateboard over one of the ramps, falls and suffers serious injuries. Is the construction company liable for the 12-year-old's injuries?(A) No, because the 12-year-old boy was a trespasser.(B) No, if the 12-year-old boy was warned of the danger.(C) Yes, if the construction company could have removed or disassembled the ramps at little cost. (D) Yes, because the construction company created a public nuisance.

The correct answer is: (C) Yes, if the construction company could have removed or disassembled the ramps at little cost. In negligence actions for damages arising from artificial conditions on a defendant's land, a heightened standard of care may apply regarding child trespassers. If the defendant knows that children are trespassing, the defendant has a duty to exercise reasonable care to prevent injury to trespassing children arising from dangerous artificial conditions. The burden of eliminating the dangerous artificial condition will be weighed against the risk to trespassing children. Thus, here, if the construction company could have removed or disassembled the ramps when work was not underway at little cost, the construction company will be liable for the 12-year-old boy's injuries.

A woman was injured when she slipped and fell at a local hardware store. She approached an attorney in the area about representing her in her suit against the hardware store. The attorney quoted his fee to the woman, who agreed to pay that amount. The woman and the attorney signed a contract detailing that he would represent the woman in her personal injury lawsuit against the hardware store. After a period of months during which the woman received no communication from the attorney, the woman discovered that the attorney had never filed the suit. The woman also discovered that the statute of limitations had run on her suit and that she could no longer recover from the hardware store for her injuries. The woman then filed suit against the attorney for malpractice. In addition to proving that the attorney was negligent in failing to file the lawsuit against the hardware, the woman must prove, at a minimum, that (A) she was not at fault for the failure to file the lawsuit. (B) she was severely injured in the fall in the market. (C) but for the attorney's negligence, she would have recovered damages from the hardware store. (D) she had a good faith claim against the hardware store that was lost due to the attorney's misconduct.

The correct answer is: (C) but for the attorney's negligence, she would have recovered damages from the hardware store. To prevail in a negligence action, the plaintiff must show that the defendant's act caused her actual damages. Thus, in this case, the woman can recover against the attorney only if she can show that she would have proceeded with and won the underlying action against the hardware store if the attorney had not been negligent. If she would have lost the underlying suit, the attorney's negligence caused her no loss.

Homeowner lived in a neighborhood of single-family homes on large lots with mature trees. Homeowner hired a tree surgeon to inspect his property and remove any diseased or damaged trees, and also trim any diseased or dead branches on the otherwise healthy trees. Next door to Homeowner lived a family with a ten-year-old daughter who loved to climb trees. There was a very tall oak tree that was located on Homeowner's property, but a few branches overhung the property of the girl's family. If she climbed to the top of the jungle gym on her family's property, she could reach the lowest branch of the oak and, from there, ascend very high up the tree. She loved to do this, and although her family worried, they did not forbid her from climbing the tree nor did they insist she come down when they saw her in the high branches of the oak. One day, as she was climbing the tree, the branch she reached for broke and she fell into Homeowner's property, landing on the roof of the detached garage that had just been built a year ago. She suffered several broken bones and internal injuries but recovered fully, although she was hospitalized for several weeks. When she fell, the impact damaged the garage, necessitating repairs to the garage roof. The girl's parents filed suit against both Homeowner and the tree surgeon. Homeowner, in turn, sued the tree surgeon, the girl's parents, and the contractor who built the garage for the damage to his garage's roof. It is revealed through discovery that the tree surgeon neglected to inspect the higher branches of oak tree the girl was climbing, because the lower branches were healthy and sound. Had the tree surgeon inspected the branch, he would have seen that the branch was dead and needed to be trimmed. Moreover, discovery reveals that the contractor did not properly frame the garage, leaving it more vulnerable to damage from falling objects. Against which defendant, if any, is Homeowner likely to prevail? (A) Homeowner will prevail against the tree surgeon only. (B) Homeowner will prevail against the tree surgeon and the contractor only. (C) Homeowner will not prevail against any defendant. (D) Homeowner will prevail against all defendants.

The correct answer is: (D) Homeowner will prevail against all defendants. A plaintiff seeking to establish liability in a negligence action must show the following: that the defendant owed a legal duty to plaintiff; that the defendant breached that duty; that the defendant's breach of duty was the actual and legal (proximate) cause of injury to the plaintiff; and that the plaintiff suffered damages as a result of the defendant's conduct. Here, Homeowner can make out a prima facie case for negligence against the tree surgeon, for failing to inspect the higher branches of the oak, and against the contractor, for failing to properly frame the garage. Homeowner also is likely to recover against the child's parents. Generally, a parent is not vicariously liable for the tortious conduct of his child. However, most jurisdictions impose vicarious liability where (1) the parent entrusts the child with a dangerous instrumentality; (2) the child engages in tortious conduct while acting as a servant or agent of the parent; (3) the parent knows of the child's wrongdoing and consents to it; or (4) the parent fails to exercise proper control over his child. Here, the facts indicate that the girl's parents failed to exercise proper control over her, by failing to forbid her, or prevent her, from climbing high in the oak tree.

City sewer workers fixing a public sewage line on the defendant's property observed several marijuana plants and advised police. The property was surrounded by a wall preventing the officers from corroborating the tip. Officers used a police helicopter to view the inside of the yard and observed the plants and used the information to obtain an arrest warrant. The defendant moves to quash her arrest based on the aerial observations of the marijuana used to obtain the warrant. Should the court quash the arrest? (A) Yes, because the defendant had a reasonable expectation of privacy in a walled off garden from public view. (B) Yes, because there was a reasonable expectation of privacy in the curtilage surrounding the house which included the garden. (C) No, because the defendant did not have a reasonable expectation of privacy in a walled off garden from public view. (D) No, because aerial observation does not violate the defendant's reasonable expectation of privacy.

The correct answer is: (D) No, because aerial observation does not violate the defendant's reasonable expectation of privacy. Police officers received a reliable tip from city workers that the defendant was growing marijuana in the defendant's yard which was surrounded by a wall to ensure privacy. There is no reasonable expectation of privacy from police observations during a flyover. The defendant may have a reasonable expectation of privacy that the property will not be observed from the ground, but that expectation does not extend to aerial searches.

A merchant owned a large ranch in the western United States. The merchant promised a ranch foreman that he would receive the ranch when the merchant died. However, the merchant's will left everything to the merchant's niece. The niece lived in New York and never visited the ranch. She refused to respond personally to a letter from the foreman. The niece had the executor of the merchant's estate shut down the ranch, fire the ranch foreman, and inform the ranch foreman that the ranch would never be his. Two years later, the ranch foreman returned to the land and reopened the ranch. The ranch foreman paid the property taxes and otherwise held himself out as the owner of the land for the next 21 years. The ranch foreman finally decided that he was getting too old to work the ranch, and he contracted for the sale of the land to a conservation group. Their contract required that the ranch foreman convey "good and marketable title." Before delivering the purchase price, an attorney for the conservation group discovered that the title to the land was in the niece's name, not the foreman's name, and the conservation group refused to go forward with the purchase. The ranch foreman sued the conservation group for breach of contract. The relevant statutory period is 20 years. Will the ranch foreman prevail against the conservation group? (A) Yes, because the ranch foreman is entitled to specific performance of the purchase and sale contract. (B) Yes, because the ranch foreman is entitled to damages from the conservation group for breach of contract. (C) No, because the niece, as the owner of the ranch, is entitled to enforce the contract and receive payment from the conservation group. (D) No, because although the ranch foreman is the owner of the ranch, the foreman's title is not marketable.

The correct answer is: (D) No, because although the ranch foreman is the owner of the ranch, the foreman's title is not marketable. The ranch foreman is the owner of the ranch by adverse possession because his possession was open, visible, and notorious; actual; exclusive; hostile; and continuous for the statutory period. Title acquired by adverse possession may become marketable if the adverse possessor quiets title before attempting to convey it. However, because there is no evidence that the ranch foreman had brought an action to prove his adverse possession claim and quiet title, the ranch foreman's title was not marketable. Modern courts might still find that title acquired by adverse possession is marketable if the adverse possession has been for a very long time; the record owner is unlikely to sue; and the record owner is unlikely to prevail if she did. The facts do not suggest that the niece, as title holder of record, would be unlikely to sue. Therefore, the ranch foreman would not prevail.

A husband and wife were domiciled in State A. The couple had marital assets (net of all liabilities) that were valued at $250,000. One of the significant assets was an account that contained United States Savings Bonds. The husband decided that he wanted a divorce and moved to State B, with the intent to remain there indefinitely. The wife did not move. The wife sued the husband for divorce in the United States District Court for State A and alleged that she was entitled to one-half of the marital estate, $125,000. The husband moved to dismiss the case for lack of subject matter jurisdiction. The court denied the motion. Was the court correct to deny the motion to dismiss for lack of subject matter jurisdiction? (A) Yes, because the case concerned ownership of an asset created under federal law. (B) Yes, because the husband became a citizen of State B before the case was filed. (C) No, because the husband was a citizen of State A before the case was filed. (D) No, because federal courts do not have subject matter jurisdiction over divorce cases.

The correct answer is: (D) No, because federal courts do not have subject matter jurisdiction over divorce cases. The parties to this case were of diverse citizenship when it matters the moment of the filing of the case and more than $75,000 was in controversy. Nevertheless, the case fell within the well-established "domestic relations exception" to federal subject matter jurisdiction. Despite the literal terms of the diversity statute, federal courts do not have jurisdiction over divorce cases, regardless of the citizenship of the parties or the amount in controversy.

A computer store places an advertising circular into the local newspaper and advertises that it will sell tablet computers (normally $490.00) for "$.390.00" to the "first fifty customers to come to the store on Wednesday." The first customer to arrive at the store tenders his check for $.39 to the store, which refuses to sell him the tablet computer. The store tells the customer that the price in the circular was a misprint. May the customer enforce a contract to sell the tablet computer for the advertised price? (A) Yes, because the advertisement was an offer, and the customer unequivocally accepted the offer when he tendered his check. (B) Yes, because the "first fifty customers" language made the advertisement an offer rather than an invitation for offers. (C) No, because the advertisement was not an offer but rather merely an invitation to make offers. (D) No, because no reasonable person would believe that the offer to sell the tablet computer for "$.390.00" was anything other than a misprint.

The correct answer is: (D) No, because no reasonable person would believe that the offer to sell the tablet computer for "$.390.00" was anything other than a misprint. This answer is correct. The advertisement came much closer to being an offer when the store included the "first fifty customers" language. Communication of an offer to multiple offerees is generally an invitation for an offer rather than an offer unless that communication indicates how goods will be allocated in the event of excess demand, such as by limiting the offer to "first come, first serve" or a certain number of customers. However, an offer that is too good to be true and which a reasonable person would not believe to be a valid offer, is not an offer. In this case, because no reasonable person would believe that the store would sell tablet computers for 39 cents, particularly by using an advertisement that stated the price in such a nonsensical way, as "$.390.00," this language does not constitute an offer.

A veterinarian was driving home along a country road in the evening after vaccinating several head of cattle. The road had many potholes and ruts, and after bumping through an especially deep hole, she heard a loud clank. She pulled over to inspect her wheels and found that a hubcap had come loose. She kicked the hubcap to snap it back into place and then got back into her car and proceeded on her way home. A few minutes later, a fox darted out in front of her car. Slamming the brakes to avoid the fox, her hubcap flew off and hit another car traveling behind her. The driver of the trailing car sued the veterinarian for the damage to his vehicle. The veterinarian claims she is not at fault; is she correct? (A) Yes, because she could not have foreseen that the hubcap would fall off. (B) Yes, because it was not her fault that the road was poorly maintained. (C) No, because she should have hit the fox. (D) No, because she knew that the hubcap could come off.

The correct answer is: (D) No, because she knew that the hubcap could come off. A defendant is considered to have breached his duty of reasonable care when he fails to conduct himself as a reasonable person would under the same circumstances. If the burden of taking precautions is less than the probability of injury, the defendant has breached his duty to the plaintiff. Here, the veterinarian knew that her hubcap had come loose earlier that evening, and yet she kicked it back on, rather than putting it in her vehicle until it could be professionally tightened. She also knew that the road was poorly maintained, and yet she slammed on the brakes as she was driving. Therefore, the veterinarian is liable to the driver for the damage her hubcap caused to his car.

A hall monitor at a public school observed students congregating around the cheerleader's locker. The monitor observed several students handing the cheerleader cash in exchange for brown paper lunch bags. The hall monitor advised the Principal of the suspicious activity. When confronted by the Principal, the cheerleader denied that the activity was unlawful and claimed that she was selling cookies to the other students. The Principal ordered the cheerleader to open the locker and hand him one of the brown paper lunch bags. Inside were five rolled marijuana cigarettes. The cheerleader was later charged by police with possession and intent to distribute drugs. Her lawyer filed a motion to quash the arrest and suppress evidence. Should the court grant the motion? (A) Yes, because the Principal did not have probable cause to search the locker. (B) Yes, because the search violated the cheerleader's reasonable expectation of privacy. (C) No, because the Principal was not acting as an agent of the police at the time of the search. (D) No, because the Principal had a reasonable suspicion based on the hall monitor's observations that the cheerleader was selling drugs out of her locker.

The correct answer is: (D) No, because the Principal had a reasonable suspicion based on the hall monitor's observations that the cheerleader was selling drugs out of her locker. Public school officials only need reasonable grounds to justify a student search. Neither a warrant nor probable cause is required. Based on the observation of cash in exchange for the delivery of a brown paper bag from inside of a locker, the school officials had reasonable grounds to examine the contents of the bags and the locker.

Driver was arrested after running a stop sign and driving with an expired license. The driver's cousin was seated in the front passenger seat. The police officer searched the entire passenger compartment at the time of the arrest and observed baskets of fruit in the back seat. When asked about the source of the fruit, the driver explained that they had just left an apple orchard and that crates of vegetables also purchased at the orchard were loaded in the trunk. The officer then searched the trunk and observed the vegetables along with two bricks of marijuana. The driver and cousin were both charged with drug offenses. At trial, the court granted the driver's motion to suppress the evidence located in the trunk. Should the court grant the cousin's motion to suppress the same evidence? (A) Yes, based on the principle of collateral estoppel. (B) Yes, because the marijuana was illegally seized from the trunk. (C) No, because the police officer had probable cause to search the trunk after he located the fruit in the back seat. (D) No, because the cousin had no standing to object to the search.

The correct answer is: (D) No, because the cousin had no standing to object to the search. The driver owned the vehicle and therefore, as the owner, had standing to contest the legality of the search. However, the cousin had no personal interest in the automobile and lacked standing to contest the search. The officer lacked probable cause to search the trunk based on observing the basket of fruit in the back seat. The search of the trunk was illegal as the driver did not consent to the warrantless search and no warrant exception applied based on these facts. The evidence will be suppressed as to the driver but will remain admissible against the cousin.

A teenage brother and sister had heard that a locally notorious immigrant, who had become extremely wealthy through a number of ethically questionable business ventures, did not trust American banks and kept all his money in a suitcase in his garage. One night, the teenagers decided to sneak into the man's garage and steal the suitcase full of cash. They jumped the fence surrounding his home and attempted to jimmy the garage door. They had just managed to enter the garage when his dog, alerted by the noises coming from the garage area, began barking furiously, causing the man to come out to investigate. In a panic, the brother grabbed a bicycle that was sitting in the garage and beckoned his sister to get on. They rode away as fast as they could, with the sister perched on the bike's handlebars, as the man chased after them yelling for help. As they turned a corner, however, they were met by a policeman who heard the commotion. Sensing that a crime had been committed, he ordered them to stop, but the brother kept pedaling. The policeman fired a shot at the tire of the bicycle in order to stop the fleeing teenagers, but the bullet struck and killed the sister. In a jurisdiction applying the minority Redline limitation, can the brother be charged for his sister's death? (A) Yes, for felony murder, because they committed a burglary. (B) Yes, for involuntary manslaughter, because the brother was negligent in causing his sister's death. (C) No, because they had already left the scene of the felony. (D) No, because the deceased was a co-felon.

The correct answer is: (D) No, because the deceased was a co-felon. Felony murder is an unintentional killing proximately caused during the commission or attempted commission of a serious or inherently dangerous felony, such as burglary, arson, robbery, rape, or kidnapping ("BARRK"). For a defendant to be guilty of felony murder, the resulting death must be a foreseeable outgrowth of the defendant's actions. Courts have generally applied the foreseeability requirement very liberally, and for purposes of felony murder, most deaths are considered foreseeable. If a death occurs while the defendant is fleeing from the scene of the felony, he may still be guilty of felony murder. However, under the Redline limitation, a felon is not guilty of felony murder where the killing constitutes a justifiable homicide, such as where the police or the victim shoot one of the co-felons. In this case, inasmuch as a policeman shot the sister, his co-felon, the brother will not be criminally liable for his sister's death.

Plaintiff's and defendant's lawyers meet face to face to discuss settlement of a pending lawsuit. At the beginning of the conversation, defendant's lawyer offers to settle the case for the payment of $10,000 by defendant in exchange for a signed release from the plaintiff. The defendant's lawyer tells plaintiff's lawyer that this offer "is only good for 24 hours." The lawyers negotiate further but no additional offers are made. Twelve hours later, the defendant's lawyer calls plaintiff's lawyer on the phone and tells him that the settlement offer is withdrawn. One hour later, the plaintiff's lawyer shows up in the defendant's lawyer's office with a signed release from plaintiff and demands the check for $10,000. The defendant's lawyer refuses. Is the promise to settle the case for $10,000 enforceable? (A) Yes, because the plaintiff's lawyer accepted the offer within the 24-hour time limit. (B) Yes, because the plaintiff's lawyer accepted the offer within a reasonable time. (C) No, because the plaintiff's lawyer did not accept the offer before the end of the face-to-face conversation with defendant's lawyer. (D) No, because the defendant's lawyer revoked the offer before the plaintiff's lawyer accepted it.

The correct answer is: (D) No, because the defendant's lawyer revoked the offer before the plaintiff's lawyer accepted it. The defendant's lawyer indicated the offer was "good for 24 hours," but the general rule is that an offeror may revoke the offer at any time for any reason as long as the revocation occurs prior to acceptance and is effectively communicated. Because the defendant's lawyer told plaintiff's lawyer personally that the offer was revoked, before the release was presented, both of those conditions are met here. Thus, the plaintiff's acceptance came too late, after revocation. [Note: Make sure you understand the timeline. Choices B and C show how difficult it is to write good distractor.]

On May 1, an attorney received a job offer from a law firm for an annual salary of $100,000 to begin August 1, and requesting a response to the offer by the end of the month. On May 25, the attorney sent an email to the managing partner at the law firm stating: "Would you consider $110,000 as my salary? I cannot begin until October 1." The partner responded: "We really need someone to begin in August, and $100,000 is the best we can do." On June 5, the partner had not heard back from the attorney, and the law firm extended an offer to another candidate, who accepted immediately. On June 12, the attorney emailed the partner: "I accept the offer for $100,000 to begin August 1." Did a contract form between the attorney and the law firm? (A) Yes, because the partner never revoked the offer for $100,000 and beginning August 1. (B) Yes, because the partner revived the offer after the attorney counter-offered. (C) No, because employment contracts are at-will, which the law firm was free to terminate at any time for any reason. (D) No, because the offer lapsed.

The correct answer is: (D) No, because the offer lapsed. An offer terminates by lapse of time when either the time specified is the offer expires, or if no time is specified in the offer, a reasonable time. Here, the offer stated that a response was needed by the end of the month (May). On May 25, the attorney communicated with the partner. The attorney's question regarding salary was likely only "testing the waters," but the statement about the start date likely constituted a counteroffer because the attorney expressly said he/she could not begin until October. Whether the attorney's communication was a counteroffer or only testing the waters, the partner's response indicated that the offer for $100,000 to begin August 1 was still open. However, the attorney then did not respond again until June 12. A response was needed by the end of May, and the power of acceptance lapsed when the attorney did not respond by the end of the month. The attorney no longer had the power of acceptance on June 12, so a contract did not form.

A corporation was organized under the laws of Nation A, a foreign country, and had its principal place of business in Nation A. Because of the breakdown of centralized authority in Nation A, the president of the corporation moved to State A and conducted all of the affairs of the corporation in State A, with the intent to return to Nation A and conduct the affairs of the corporation there when centralized authority was restored. A plaintiff sued the corporation in federal court in State A for a cause of action that arose entirely in Nation A and did not in any way relate to or arise out of any of the corporation's contacts with State A. State A's long-arm statute extended to the limits of the Due Process Clause of the United States Constitution. The corporation filed a timely motion to dismiss for lack of personal jurisdiction. The court denied the motion. Did the court err in denying the motion to dismiss? (A) Yes, because the claim did not arise out of or relate to any contacts with the forum state. (B) Yes, because the corporation was not incorporated in State A and maintained its principal place of business in State A only on a temporary basis. (C) No, because State A's long-arm statute extended to the limits of the Due Process Clause of the United States Constitution. (D) No, because there was general jurisdiction over the corporation in State A.

The correct answer is: (D) No, because there was general jurisdiction over the corporation in State A. The facts of this question closely track the facts of Perkins v. Benguet Mining Corp., in which the Supreme Court of the United States held that there was general jurisdiction in Ohio over a foreign corporation that was temporarily conducting all of its affairs in Ohio because of the Japanese occupation of the Philippines. In Daimler AG v. Bauman, the Court recently reaffirmed that Perkins was decided correctly and that it represented one of the few circumstances under which a state could exercise jurisdiction over claims that have no connection to the state.

A city was experiencing a severe water shortage. To mitigate the problem, the city enacted an ordinance limiting total water use and prohibiting the use of water by private citizens for any purpose other than that of personal hygiene, cooking and personal consumption. Use of water for landscaping, gardening, washing cars and the like was strictly prohibited and was punishable by fines of up to $5,000. A homeowner had a thriving vegetable garden which required daily watering. Since she limited her personal water use, she felt justified in limited use of sprinklers to water her garden. The sprinklers were timed to water the garden between 6:00 and 6:20 p.m. every night, before the homeowner got home from work. As her garden was in her backyard, the homeowner was relatively confident that her water use would go unnoticed by town officials, and it did. However, the sprinklers did not escape the attention of a boy and his friend, the seven- and eight-year-old boys who lived next-door to the homeowner. Noting that the homeowner's sprinklers went off like clockwork every night at 6:00 p.m., just when temperatures in the desert town were peaking, the boy and his friend made a habit of putting on their bathing suits and sneaking into the homeowner's backyard to run through the sprinklers, unbeknown to their mother and to the homeowner. One evening, in his enthusiasm, the boy, tripped over the sprinkler spigot and scratched his ankle. The wound went unnoticed by his mother until several days later, when she noticed that her son was limping. She took him to the doctor, only to learn that he had developed a staph infection and equired immediate hospitalization. The boy was hospitalized for over a month before he recovered sufficiently to be released, but his infected leg remained weak and his medical bills were extensive. The boy's mother filed suit against the homeowner to recover her medical expenses and lost wages, citing the homeowner's violation of the ordinance. The court accepted the statute as written. Which of the following statements is most accurate? (A) The boy's mother will prevail, if she can show causation and damages stemming from the homeowner's violation of the ordinance constitute negligence per se. (B) The boy's mother will prevail, under the attractive nuisance doctrine. (C) The boy's mother will not prevail, because she was negligent in failing to control her children. (D) The boy's mother will not prevail on grounds of negligence per se.

The correct answer is: (D) The boy's mother will not prevail on grounds of negligence per se. The unexcused violation of a legislative enactment or administrative regulation which is adopted by the court as defining the standard of conduct of a reasonable man, is negligence itself, or as commonly phrased, negligence per se. The violation of a criminal statute constitutes negligence per se where the plaintiff can show that: 1) he was a member of the class of persons the statute was designed to protect; and 2) he suffered the type of harm the statute was designed to prevent. In such instances, statutory violation will be considered sufficient to prove duty and breach of duty, but a plaintiff is still required to show causation and damages. Here, the doctrine of negligence per se does not apply. For one, the statute involved was a civil ordinance, not a criminal statute. And secondly, the type of harm suffered by the boy (an injury occurring when tripping over a sprinkler spigot) was not type of harm that the ordinance was designed to prevent. As such, the boy's mother will not prevail on grounds of negligence per se.

A fraternity president lived in a fraternity house with 25 other fraternity brothers. The fraternity had been known for years for its weekly wild keg parties. One of the better known features of the party was that at the end of the party all of the empty beer kegs were stacked up one on top of the other as evidence of the fraternity brothers' drinking prowess. This was a fraternal tradition. Every new fraternity brother was required to help the president stack the empty kegs. This week a new brother accepted the honor of stacking the kegs. The next morning the kegs came crashing down on a mailman who delivered mail to the fraternity house, breaking several bones in his foot. The mailman sued the new brother. Which if the following is correct? (A) The brother should not be held liable, because he was acting consistently with a fraternity custom that had gone on every week for years. (B) The brother should not be held liable, if he was drunk when he stacked the kegs. (C) The brother should be held liable, if all fraternities follow the keg stacking custom. (D) The brother should be held liable, even though he was acting in a manner consistent with a fraternity custom that had gone on every week for years.

The correct answer is: (D) The brother should be held liable, even though he was acting in a manner consistent with a fraternity custom that had gone on every week for years. Although acting according to a custom can be a defense to a charge of negligence, it is not conclusive. The trier of fact may find either that the customary manner of behavior was not reasonable under the circumstances or that a reasonable person would not have engaged in customary behavior. A trier of fact will probably find both that the custom of stacking kegs on top of one another was not reasonable and that a reasonable person would not engage in such dangerous behavior.

Two brothers decided to open a dry cleaning business. After the brothers had been running the business for several months, they realized that although the volume of business exceeded their expectations, they were barely breaking even. Investigating further, the brothers determined that the cash registers and bank accounts did not contain the expected amount of funds. They installed a video camera in the shop and quickly discovered that an employee was stealing money. The video also included footage of the employee physically threatening a customer and stealing his clothes. The brothers immediately fired the employee. If the customer the ex-employee stole from and threatened sues the brothers, what is the likely outcome of the customer's lawsuit? (A) The brothers will be liable only for the customer's clothing, because there is no way to prove how much money the ex-employee stole from the customer. (B) The brothers will be liable only for the customer's clothing, because there is no way to prove how much money the ex-employee stole from the customer. (C) The brothers will bear no liability for the ex-employee's acts, because the brothers terminated the employee/employer relationship upon discovery of his acts. (D) The brothers will bear no liability for the ex-employee's acts, because the ex-employee did not commit the acts against the customer for the benefit of the brothers.

The correct answer is: (D) The brothers will bear no liability for the ex-employee's acts, because the ex- employee did not commit the acts against the customer for the benefit of the brothers. An employer is liable for the tortious acts of his or her employees that are committed within the scope of employment and that cause injuries or property damage to a third person. The "scope of employment" includes acts so closely connected and reasonably incidental to what the servant was employed to do that they may be regarded as methods, however improper, to carry out the employer's objectives. Here, the ex-employee's actions amount to an intentional tort. However, because the actions were not committed to further the brothers' business objectives, the brothers will bear no liability for the ex- employee's actions.

A company that made heating units was a State X partnership composed of an engineer and a machinist. The company manufactured and sold heating units for use in apartments. The company's only manufacturing plant, warehouse, and business office was in State X. Three months ago, a landlord of a large apartment in State Z bought forty heating units to be installed in the landlord's apartments. The engineer and the machinist were domiciled in State X. Two weeks ago, a tenant was severely burned when a heating unit malfunctioned and caused a fire. The tenant sued both the company and the landlord in the U.S. District Court in State Z. The engineer and the machinist were personally served in State X. In determining if the assertion of personal jurisdiction over the company is proper, which of the following factors is least pertinent? (A) The State Z long-arm statute. (B) Whether the company was aware where the heating unit would be used. (C) Whether the company presently does any other business in State Z (whether or not that business is related to the sale to the landlord). (D) The physical health of the engineer and the machinist.

The correct answer is: (D) The physical health of the engineer and the machinist. The physical health of the parties involved in litigation has never been cited as a relevant factor in the determination of whether personal jurisdiction exists or not (it could presumably be considered in the context of a forum non conveniens motion).

While on his evening foot patrol, an officer heard loud music coming from a basement apartment. The curtains were drawn except for a two inch opening. The officer walked into the stairwell leading to the door and observed the renter and her boyfriend inhaling cocaine through a plastic straw. The officer announced his presence and demanded that the renter open up the door. The renter opened the door and was immediately arrested along with her boyfriend on drug charges. At her trial, the renter moves to suppress evidence of the cocaine. Should the court grant the motion? (A) No, because the entry to the apartment constituted a valid consent search. (B) No, because the search was conducted incident to a valid arrest. (C) No, because the drugs were in plain view. (D) Yes, because none of the exceptions to the warrant requirement apply.

The correct answer is: (D) Yes, because none of the exceptions to the warrant requirement apply. The renter has a reasonable expectation of privacy in her residence. Officers can only look into a residence if the officer is standing in a place where the defendant does not have a reasonable expectation of privacy. Officers may not stand in the curtilage and peer into a narrow slit in the curtains to observe criminal behavior and then demand entry into the residence. This is not plain view. Unless a well- established exception to the warrant requirement applies, officers must obtain a warrant based upon a finding of probable cause in order to cross the threshold of a residence.

A technical support employee of a computer company spent his morning traveling the city providing in- home tech support to multiple customers. The last scheduled home repair visit took much longer than normal, and the employee telephoned the office to ask his boss whether he would mind if the employee went home for lunch rather than return to the office before making his final series of tech support calls in the late afternoon. The employee's boss agreed, and the employee drove home to eat lunch. While at home, the employee drank several beers with his meal. Realizing he was too intoxicated to drive, the employee had his precocious 10-year-old daughter drive him to his next tech support call. The daughter drove him safely across town but, upon entering the client's driveway, she crashed into the client's car. Will the computer company be liable for the accident? (A) No, because the scope of the employee's employment was tech support, not driving an automobile. (B) No, because the daughter was not an employee. (C) Yes, because the scope of the employee's employment includes in-home tech support. (D) Yes, because the employee's negligence will be imputed to his employer.

The correct answer is: (D) Yes, because the employee's negligence will be imputed to his employer. An employer is liable for his or her employees' tortious acts that are committed within the scope of employment and that cause injuries or property damage to a third person. If an employee negligently entrusts performance of his work to a third person, the employer may be liable by imputed negligence. In this case, it was the daughter who was driving the car, and there is nothing to indicate that she was an employee of the computer company. However, given that the employee breached his duty of due care by having his young daughter drive him to his next service call, the employee's negligence may be imputed to his employer, and the computer company will be liable for the resulting damages. As such, this is the best answer.

In 1989, Oliver, the owner of an estate, was adjudicated legally incompetent. In 1990, Albert entered adversely on land owned by Oliver. In 1993, Oliver died without regaining legal competence; however, Oliver left a valid will that devised his entire estate to Betty, who was under no disability at the time. The jurisdiction has a statute of limitations for adverse possession, which provides that, in the case of disability, the statutory period expires seven (7) years after the adverse possessor enters or ten (10) years after the owner's disability is removed, whichever date is later. In 1998, Betty sued Albert to quiet title in the estate. Should the court rule for Betty? (A) No, because Albert acquired title by adverse possession. (B) No, because Betty was not in privity of estate with Oliver. (C) Yes, because Betty is the rightful owner of the estate. (D) Yes, because the statute of limitations has not run.

The correct answer is: (D) Yes, because the statute of limitations has not run. The running of a statute of limitations for adverse possession may be tolled in certain circumstances. For tolling to occur, a disability must be in existence at the time the adverse possessor enters. Disability typically includes age, legal incompetence, or imprisonment. Here, the statute tolled because Oliver was under a disability at the time that Albert entered the premises. Oliver's disability was removed when he died in 1993. The statute gives Betty, as the successor owner, who was in privity of estate with Oliver through Oliver's will, 10 years after the removal of Oliver's disability. Thus, the statute would run against Betty in 2003, and Betty should prevail in her 1998 suit.

On August 1, a manufacturer of portable drinking fountains mailed to the retailer of drinking fountains the following written offer: "Will sell 100 drinking fountains at our list price of $50 each. Available for immediate delivery. Please respond by return mail." Unaware of the manufacturer's offer, the retailer, also on August 1, mailed a written letter to the manufacturer containing the identical terms: "Will purchase 100 drinking fountains at your list price of $50 each." Both letters were mailed through the U.S. postal system. Before either party has received the other's correspondence, which of the following accurately states the legal relationship between the manufacturer and retailer? (A) Since there was obviously a "meeting of the minds," a contract was formed when the retailer mailed its letter. (B) A contract exists with U.C.C. gap filler provisions controlling for delivery and payment terms. (C) A contract was formed when the retailer deposited its "acceptance," since the cross communications contained identical terms. (D) No contract exists because a posted offer has no legal effect.

Up to one-third of the Contracts questions will cover formation. This issue involving "cross offers" is heavily tested. The first requirement of a contract is that the parties manifest their mutual acknowledgment of the same terms. Normally, this will take the form of an offer and acceptance. Knowledge of the offer by the offeree is essential to acceptance. In cases involving cross offers, neither offer accepts the other. Since neither party has expressed a willingness to be bound by the other's terms, no contract exists despite the fact that the cross offers contain identical terms. [Note: This is a "sexy answer" question because everyone wants there to be a contract. Both parties clearly wanted the same thing. It makes sense that two people who want the same thing should be in a contract. But you need to find a way to justify that there is a valid contract, i.e. offer and acceptance. Here, there was no meeting of the minds since they did not know about each other's offer and never accepted. Thus no contract.] The correct answer is: (D) No contract exists because a posted offer has no legal effect.

An uncle told his nephew: "If you mow my lawn by the end of the week, I'll give you $20." The nephew responded that he was not sure if he would have time. The next day, the nephew went to his uncle's house and began mowing the lawn. When the nephew had mowed about half the lawn, the lawn mower ran out of gas. The nephew did not complete the job. Is the uncle obligated to pay the nephew $20? (A) No, because the nephew abandoned performance and did not accept the offer. (B) No, because the offer was not in writing. (C) Yes, because the nephew accepted the offer by beginning performance. (D) Yes, because the uncle could not revoke the offer after the nephew began to perform.

[Note: A unilateral contract is accepted upon completion of performance. Be careful with facts and do not confuse acceptance with revocation of a unilateral contract, see D.]. The correct answer is: (A) No, because the nephew abandoned performance and did not accept the offer. This question is an example of a unilateral contract whereby the uncle is seeking the nephew's performance. With a unilateral contract, the offeror is bound only when the offeree completes performance; in other words, acceptance is only effective upon completion of performance. The nephew only mowed half of the lawn. Because the nephew did not complete performance, the nephew did not accept the offer, and the uncle is not obligated to pay the nephew $20.

A tourist visits an arts and craft store in a mountain community. The tourist decides that he likes a large sculpture that sits in front of the store. When the tourist gets home, he writes the store owner a letter and sends her a proposed contract under which the tourist will pay the store owner $3,000 to create a similar sculpture for the tourist's home. The store owner signs the contract and immediately mails it to the tourist. The next day, before the tourist receives the signed contract from the store owner, the tourist calls the store owner to say that he has changed his mind and does not want the store owner to do the sculpture. Do the tourist and the store owner have a binding contract for the store owner to create the sculpture? (A) Yes, because the store owner signed the contract before the tourist called to revoke the offer. (B) Yes, because the store owner mailed the signed contract before the tourist called to revoke the offer. (C) No, because the tourist revoked the offer before he received the signed contract from the store owner. (D) No, because the tourist revoked the offer before the store owner took any action in reliance upon the tourist's promise to pay her $3,000 for the sculpture.

[Note: Classic mailbox rule fact pattern, i.e. an acceptance is valid when mailed.] The correct answer is: (B) Yes, because the store owner mailed the signed contract before the tourist called to revoke the offer. This answer is correct because it is a straightforward application of the "mailbox rule" which states that acceptance of an offer by mail is effective upon dispatch. The store owner's acceptance of the offer was effective the moment she mailed the contract back to the tourist. The tourist's later purported revocation of the offer has no effect because the store owner has already accepted the offer.

A customer went to a home improvement store to purchase blinds for the windows in the customer's home. The customer ordered the blinds for $600, which were to be installed by the home improvement store the following week. Why does the UCC apply to this transaction? (A) Because the UCC applies to all sales of goods and services. (B) Because the service of installing the blinds was incidental to the sale of the blinds. (C) Because the sale of the blinds was priced over $500. (D) Because the home improvement store is a merchant.

[Note: This fact pattern can also be answered using a process of elimination.] The correct answer is: (B) Because the service of installing the blinds was incidental to the sale of the blinds. The UCC applies to sales of goods; the UCC does not apply to transactions for services. The blinds are a good, but the installation of the blinds is a service. Where both the sale of goods and a services contract arise together, the predominant purpose of the transaction governs whether the UCC or common law applies to the contract. Here, the predominant purpose of the contract is the sale of the blinds, to which the installation is incidental. Thus, the UCC applies.

A woman had dreams of moving to State X to become an actress. For the past 4 years she rented an apartment with a month to month lease and worked as a bartender in State A. She acted in local plays and was waiting to be discovered. She was driving to work one day when she had a car accident with a delivery truck owned by a corporation. The woman decided to file a claim against the corporation, claiming its driver was negligent in causing her injuries, resulting in $25,000 in damages and $60,000 in medical expenses. The corporation was incorporated in State B, and had their office where their officers direct, control, and coordinate the corporation's activities in State X. The corporation had a large delivery truck depot in State A. The woman's attorney told the corporation that the woman was planning on suing the corporation in federal court. The president of the corporation told the attorney that the corporation would have the case removed to a State X state court. Could the attorney properly file the woman's claim in federal court? (A) No, because she was only claiming $25,000 in damages. (B) No, because both the woman and the corporation were citizens of State A. (C) Yes, because the woman was only renting property in State A and intended to move to State X to pursue a career in acting. (D) Yes, because the court had original subject-matter jurisdiction.

he correct answer is: (D) Yes, because the court had original subject-matter jurisdiction. Federal district courts have original jurisdiction over matters involving diverse litigants, where the amount in controversy is more than $75,000 [28 U.S.C. Sec. 1332]. In order for diversity to exist, the dispute must involve citizens of different states. A person will be considered the citizen of his or her state of domicile. Domicile requires the person to have both physical presence in the state and the intent to remain in that state indefinitely. A corporation is a citizen of both the state of its incorporation and the state where it has its principal place of business. A corporation may have only one principal place of business, namely the corporate "nerve center," or the location where the corporation's officers direct, control, and coordinate the corporation's activities. Finally, in order to meet the $75,000 threshold, a claimant may combine each amount in controversy against a single defendant. For the reasons stated above, the woman would be considered a citizen of State A, while the corporation would be considered a citizen of both State B and State X. Additionally, the woman's claims of $25,000 in damages and $60,000 in medical expenses could be combined to meet the $75,000 requirement.

Over 25 years ago, a geologist paid a doctor $10,000 for a general warranty deed to a large rural property. Unbeknownst to the geologist, the actual owner of the property was a professor. The geologist promptly built a large house that covered the eastern half of the property. The following year, the geologist constructed a large swimming pool on land he thought was part of the parcel he had purchased. In reality, the geologist had installed the pool on land owned by a car dealer. Eight years after the geologist paid for the land and moved into his house, the professor wrote a new will leaving the property to her daughter for life, remainder to a relative. The professor died a year later. The geologist continued his possession of the property for 10 more years, when the geologist placed an ad in the paper seeking to sell the property containing the house, but not the pool. Neither the daughter nor the relative knew about the geologist's possession of the property until the geologist placed an ad in the paper. The relative confronted the geologist, and the geologist brought suit to quiet title. The statutory period for adverse possession is 15 years. Will the court award title to the geologist? (A) Yes, because the relative does not have a right to possession until after the daughter dies. (B) Yes, because the geologist has met the requirements for adverse possession. (C) No, because the geologist did not intend to acquire the property by adverse possession when he paid $10,000 to the doctor. (D) No, because the daughter became the rightful owner of the property before the running of the statutory period.

rect answer is: (B) Yes, because the geologist has met the requirements for adverse possession. In order for a party to acquire title by adverse possession, the party's possession of land must be: a) open, visible, and notorious; b) actual; c) exclusive; d) hostile and under a claim of right or title; and e) continuous for the statutory period. Here, the geologist easily meets the requirements of adverse possession. The large house satisfied the open, visible, and notorious requirement. The geologist resided on the property, without the permission of the record owner, so his possession was actual and exclusive. The geologist's possession was hostile in that it was inconsistent with the rights of the record owner and he held it under an albeit invalid deed so he had a claim of title as well. The statutory period was satisfied. The professor's subsequent division of her estate did not interrupt the running of the statutory period. Because the geologist began his adverse possession against the estate when it was a fee simple one, prior to it being conveyed as a life estate with a remainder interest, his adverse possession is against the entire estate. Thus, the geologist will most likely prevail in his action to quiet title to the property.

A driver drove his car into an intersection and collided with a fire engine that had entered the intersection from the driver's right. The accident was caused by negligence on the driver's part. As a result of the accident, the fire engine was delayed in reaching a homeowner's house, which was entirely consumed by fire. The homeowner's house was located about ten blocks from the scene of the accident. If the homeowner asserts a claim against the driver, what would the homeowner recover? (A) The part of his loss that would have been prevented if the collision had not occurred. (B) The value of his house before the fire. (C) Nothing, if the driver had nothing to do with causing the fire. (D) Nothing, because the driver's conduct did not create an apparent danger to the homeowner.

(A) is the best response, because it correctly states exactly what damages the defendant will be responsible for. You're told under these facts that the defendant was negligent. Thus, he'll be liable for any damages flowing from that negligence. Here, the plaintiff's' house was already on fire when the accident occurred; thus, that portion of the damage is not attributable to the defendant. By negligently hitting a fire truck on its way to a fire, the defendant created the foreseeable risk of loss to whoever's house the fire truck was heading. Here, the damages the defendant caused — for which he'll be liable — are the part of the loss that would have been prevented had the accident not occurred. Since A correctly states this, it's the best response.

A driver negligently ran into a pedestrian who was walking along a road. The pedestrian sustained an injury to his knee, causing it to buckle from time to time. Several months later, the pedestrian sustained an injury to his shoulder when his knee buckled, causing him to fall down a flight of stairs. The pedestrian then brought an action against the driver for the injuries to his knee and shoulder. In his action against the driver, for which of his injuries may the pedestrian recover damages? (A) For the injuries to his knee and shoulder, because the driver takes the victim as he finds him. (B) For the injuries to his knee and shoulder, if the jury finds that the pedestrian's fall down a flight of stairs was a normal consequence of his original injury. (C) For the injury to his knee only, because the injury to the pedestrian's shoulder is separable. (D) For the injury to his knee only, if the jury finds that the driver could not have foreseen that his negligent driving would cause the pedestrian to fall down a flight of stairs.

(B) is the best response, because one who negligently causes an initial injury to another is also liable for a second injury to the other that is a normal consequence of the initial injury. It's clear that the driver's negligence was the proximate cause of the pedestrian's knee injury. The challenging part of this question is that it requires you to say whether the driver is also liable for the "second" injury, the shoulder injury. This choice correctly states that the answer is "yes" if and only if the second injury is a "normal consequence" of the first injury. Rest. 2d Torts, § 460, covers precisely this situation, and applies the "normal consequence" test: "If the negligent actor is liable for an injury which impairs the physical condition of another's body, the actor is also liable for harm sustained in a subsequent accident which would not have occurred had the other's condition not been impaired, and which is a normal consequence of such impairment. Notice that it does not matter whether the driver should reasonably have foreseen that his negligent driving would or might ultimately cause this "second injury." We evaluate the second injury by looking at the facts as they existed after the first injury, and the defendant is liable if the second injury is a "normal consequence" of the first injury, no matter how unlikely the second injury may have seemed viewed as of the moment of the defendant's initial negligence.


Related study sets

Domain 1.1 Mobile Device Operating Systems

View Set

Physics Exam Vocabulary + Multiple Choice(9-10,17-19)

View Set

OSHA Quiz ( 3 ) Exit Routes, Emergency Action Plans, and Fire Prevention and Protection

View Set

Exercise 8 Overview of the Skeleton: Classification and Structure of Bones and Cartilages

View Set

Grade 7 Social Studies: Chapter 1, Meet Three of Canada's First Nations

View Set

Chapter 1: The Sciences of Anatomy & Physiology

View Set

Into to Physics Final Review (Test 2)

View Set